evaluation and management of sepsis/septic shock in adults ... › dam › aafp › documents ›...

56
1 Evaluation and Management of Sepsis/Septic Shock in Adults: 2018 Sepsis Care - Outpatient to Inpatient Victoria McCurry, MD ACTIVITY DISCLAIMER The material presented here is being made available by the American Academy of Family Physicians for educational purposes only. Please note that medical information is constantly changing; the information contained in this activity was accurate at the time of publication. This material is not intended to represent the only, nor necessarily best, methods or procedures appropriate for the medical situations discussed. Rather, it is intended to present an approach, view, statement, or opinion of the faculty, which may be helpful to others who face similar situations. The AAFP disclaims any and all liability for injury or other damages resulting to any individual using this material and for all claims that might arise out of the use of the techniques demonstrated therein by such individuals, whether these claims shall be asserted by a physician or any other person. Physicians may care to check specific details such as drug doses and contraindications, etc., in standard sources prior to clinical application. This material might contain recommendations/guidelines developed by other organizations. Please note that although these guidelines might be included, this does not necessarily imply the endorsement by the AAFP.

Upload: others

Post on 07-Jun-2020

3 views

Category:

Documents


0 download

TRANSCRIPT

Page 1: Evaluation and Management of Sepsis/Septic Shock in Adults ... › dam › AAFP › documents › events › fmx › hand… · Evaluation and Management of Sepsis/Septic Shock in

1

Evaluation and Management of Sepsis/Septic Shock in Adults:

2018 Sepsis Care - Outpatient to Inpatient

Victoria McCurry, MD

ACTIVITY DISCLAIMERThe material presented here is being made available by the American Academy of Family Physicians for educational purposes only. Please note that medical information is constantly changing; the information contained in this activity was accurate at the time of publication. This material is not intended to represent the only, nor necessarily best, methods or procedures appropriate for the medical situations discussed. Rather, it is intended to present an approach, view, statement, or opinion of the faculty, which may be helpful to others who face similar situations.

The AAFP disclaims any and all liability for injury or other damages resulting to any individual using this material and for all claims that might arise out of the use of the techniques demonstrated therein by such individuals, whether these claims shall be asserted by a physician or any other person. Physicians may care to check specific details such as drug doses and contraindications, etc., in standard sources prior to clinical application. This material might contain recommendations/guidelines developed by other organizations. Please note that although these guidelines might be included, this does not necessarily imply the endorsement by the AAFP.

Page 2: Evaluation and Management of Sepsis/Septic Shock in Adults ... › dam › AAFP › documents › events › fmx › hand… · Evaluation and Management of Sepsis/Septic Shock in

2

DISCLOSUREIt is the policy of the AAFP that all individuals in a position to control content disclose any relationships with commercial interests upon nomination/invitation of participation. Disclosure documents are reviewed for potential conflict of interest (COI), and if identified, conflicts are resolved prior to confirmation of participation. Only those participants who had no conflict of interest or who agreed to an identified resolution process prior to their participation were involved in this CME activity.

All individuals in a position to control content for this session have indicated they have no relevant financial relationships to disclose.

The content of my material/presentation in this CME activity will include discussion of unapproved or investigational uses of products or devices as indicated:

• I intend to discuss a newly-identified potential sepsis therapy that is still under investigation with a formal study, but positive early comparative results including prevention of worsening organ failure and shorter length of time on vasopressors, as detailed in the CHEST article published June 2017. Marik P, KhangooraV, et al. Hydrocortisone, Vitamin C, and Thiamine for the Treatment of Severe Sepsis and Septic Shock. Chest. 2017;151(6):1229-1238.

• An additional treatment we may discuss was just published on thiamine’s role in lowering Sepsis 28-day mortality rates. Woolum J, Abner E, Kelly A, et al. Effect of Thiamine Administration on Lactate Clearance and Mortality in Patients with Septic Shock. Crit Care Med. Published online first July 18, 2018. Issue p.

Victoria McCurry, MDDirector of Inpatient Services/Clinical Assistant Professor of Family Medicine, University of Pittsburgh Medical Center (UPMC) McKeesport Family Medicine Residency, Pennsylvania

Dr. McCurry earned her medical degree from the University of Arizona College of Medicine–Phoenix and completed her family medicine residency at the University of Arizona College of Medicine at South Campus Family Medicine Residency, Tucson, where she earned certification in integrative medicine and global health. Following residency, she was appointed as a clinical assistant professor of medicine at the University of Arizona and practiced as a hospitalist at Banner–University Medical Center South in Tucson, Arizona. She was named chair of the sepsis committee and worked to improve measurable outcomes of morbidity and mortality for septic patients. Since moving to Pittsburgh in 2017, Dr. McCurry divides her time between the outpatient clinic and the inpatient family medicine service as a clinical assistant professor of family medicine at UPMC McKeesport Family Medicine Residency. In addition, she has been chair of the residency’s quality and safety committee, and she sits on multiple hospital quality committees at UPMC McKeesport, including the sepsis committee.

Page 3: Evaluation and Management of Sepsis/Septic Shock in Adults ... › dam › AAFP › documents › events › fmx › hand… · Evaluation and Management of Sepsis/Septic Shock in

3

Learning Objectives1. Recognize the early presentation of sepsis and the tools that aid in

early detection and diagnosis.

2. Implement evidence-based protocols for initial resuscitation and infection management.

3. Establish protocols for hemodynamic support and adjunctive therapy.

4. Apply supportive therapy for severe sepsis.

5. Recognize indications for admitting a septic patient to ICU.

Audience Engagement SystemStep 1 Step 2 Step 3

Page 4: Evaluation and Management of Sepsis/Septic Shock in Adults ... › dam › AAFP › documents › events › fmx › hand… · Evaluation and Management of Sepsis/Septic Shock in

4

How to improve your sepsis management

1. Understand the significant conceptual and treatment debates ongoing with Sepsis• Sepsis 2 versus Sepsis 3• Sepsis care protocols and what those mean for clinicians

2. Elevate your clinic and hospital vigilance –‘think sepsis.’• Realize that you, the primary care provider, are often the first identifier of sepsis• Utilize clinic or hospital resources to identify sick patients and escalate treatment• Participate in peri-Sepsis care: vaccination, patient education, chronic disease management

3. Name and implement the goals of treatment for sepsis • Early broad-spectrum antibiotic• Focused fluid resuscitation, utilizing dynamic reassessment of responsiveness• In a fluid-non-responsive state vasopressors• De-escalation of antibiotic therapy • Awareness of bundle therapy and its role in hospitals, public health, and your practice

AES Question 1Please indicate your level of agreement with the following: I routinely think about sepsis when assessing my clinic patients for signs of infection, and recognize my clinic’s role in prevention of sepsis.

A. Strongly AgreeB. AgreeC. NeutralD. DisagreeE. Strongly Disagree

Page 5: Evaluation and Management of Sepsis/Septic Shock in Adults ... › dam › AAFP › documents › events › fmx › hand… · Evaluation and Management of Sepsis/Septic Shock in

5

AES Question 2Please rate your level of confidence in your ability to diagnose sepsis in your clinic office.

A.Very confidentB.ConfidentC.NeutralD.Not confidentE. Not at all confident

Not confident in Sepsis Diagnosis? • You’re not alone!

» 2016 study by Dr. Rhee et al. at Harvard» 5 clinical vignettes to evaluate for sepsis» 94 attending physicians replied to survey

• 88% critical care specialists• Median 8 years practicing• 90% at academic centers

» Poor inter-rater agreement• Five‐diagnoses (SIRS, sepsis, severe sepsis, septic shock or none) Fleiss’ 𝛘 0.29 • Dichotomized (severe sepsis including septic shock  or not) Fleiss’ 𝛘 0.23• Regardless of how data analyzed, inter‐rater agreement was poor

Rhee C, Kadri SS, Danner RL, et al. Diagnosing sepsis is subjective and highly variable: a survey of intensivists using case vignettes. Critical Care. 2016;20:89.

Page 6: Evaluation and Management of Sepsis/Septic Shock in Adults ... › dam › AAFP › documents › events › fmx › hand… · Evaluation and Management of Sepsis/Septic Shock in

6

Poster courtesy of the Centers for Disease Control and Prevention. No alternation made.

Worldwide Epidemiology• Approximately 151-31.52 million cases of sepsis

worldwide per year.– 19,400,000 cases of severe sepsis per year– 5,300,000 deaths worldwide per year2

• Financial: $24 billion US dollars1,3

– European daily sepsis patient costs €710 - €10331

• Unknown impact on low and middle income countries2

1. Seventieth World Health Assembly, April 13, 2017. “Improving the prevention, diagnosis and clinical management of sepsis” A70/13.  Agenda item 12.22. Fleischmann C, Scherag A, Adhikari NK, et al. Assessment of Global Incidence and Mortality of Hospital‐treated Sepsis. Current Estimates and Limitations. Am J Respir Crit Care Med. 2016 Feb 1;193(3):259‐73 Lagu T, Rothberg MB, Shieh MS et al. Hospitalizations, costs, and outcomes of severe sepsis in the United States 2003‐2007. Crit Care Med. 2012 Mar;40(3):754‐761.

Page 7: Evaluation and Management of Sepsis/Septic Shock in Adults ... › dam › AAFP › documents › events › fmx › hand… · Evaluation and Management of Sepsis/Septic Shock in

7

Early 2000s - Sepsis Epidemiology• Annals of the American Thoracic Society 2015 study

showed increase of 49% in sepsis claims between 2003 and 2009.1

– Supported by earlier studies2,3,4,5

• Paradoxical decline in hospital sepsis mortality rates6

• Reasons suggested: aging population, rising comorbidities,7 better care

• Most early estimates based on ‘administrative data’6,8

1. Walkey A, Lagu T, Lindenauer P. Trends in Sepsis and Infection Sources in the United States. A population‐based study. Ann Am Thorac Soc. 2015 Feb; 12(2):216‐2202. Angus DC, Linde‐ZwirbleWT, Lidicker J, et al. Epidemiology of severe sepsis in the United States: analysis of incidence, outcome, and associated costs of care. Crit Care Med. 2001 Jul;29(7):1303‐103. Martin G, Mannino D, Eaton S, Moss M. The Epidemiology of Sepsis in the United States from 1979 through 2000. N Engl J Med 2003;348:1546‐544. Lagu T, Rothberg MB, Shieh MS et al. Hospitalizations, costs, and outcomes of severe sepsis in the United States 2003‐2007. Crit Care Med. 2012 Mar;40(3):754‐761.5. Dombrovskiy VY, Martin AA, Sunderram J, Paz HL. Rapid increase in hospitalization and mortality rates for severe sepsis in the United States: a trend analysis from 1993‐2003. Crit Care Med. 2007 May;35(5):1244‐506. Stevenson E, Rubenstein A, Radin G, Wiener RS, Walkey A. Two decades of Mortality Trends among Patients with Severe Sepsis: A comparative Meta‐analysis. Crit Care Med. 2014 Mar; 42(3):625‐6317 .Singer M, Deutschman C, Seymour C, et al. The Third International Consensus Definitions for Sepsis and Septic Shock (Sepsis‐3). JAMA 2016;315(8):801‐810.8. Rhee C, Kadri S, Huang SS, et al. Objective Sepsis Surveillance Using Electronic Clinical Data. Infection control and hospital epidemiology. 2016;37(2):163‐171. 

The Centers for Disease Control and Prevention

Sepsis = medical emergency

“Doctors and nurses can prevent sepsis and also the devastating effects of sepsis, and patients and families can

watch for sepsis and ask, ‘could this be sepsis?’”1

-The Centers for Disease Control and Prevention (CDC) [Former] Director Tom Frieden, M.D., M.P.H.; August 2016

1. Tom Frieden, CDC Press Release on August 23, 2016. Viewed on 10/22/17 at https://www.cdc.gov/media/releases/2016/p0823‐sepsis‐patients.html.

Page 8: Evaluation and Management of Sepsis/Septic Shock in Adults ... › dam › AAFP › documents › events › fmx › hand… · Evaluation and Management of Sepsis/Septic Shock in

8

Poster courtesy of the Centers for Disease Control and Prevention. No alternation made.

CDC Epidemiology JAMA 2017 multi-center study, 7,801,624 adult hospitalizations1:

– In 2014, 1.7 million Sepsis hospitalizations for adults in the US• 15.6% in-hospital mortality for sepsis, or 270,000 deaths• 6% discharged to hospice• “Sepsis was present in 35% of all hospitalizations” that led to death• 10% of all US deaths due to sepsis

– Adjusted annual Sepsis incidence remained stable, from 2009 - 2014• Electronic Health Record (EHR)-based clinical data, not claims-based data

– 2009 - 2014, in-hospital mortality rates trended toward decline but lacked significance– “Discharge to hospice rates” did not change significantly

1. Rhee C, Dantes R, Epstein L, et al. Incidence and Trends of Sepsis in US Hospitals Using Clinical vs Claims Data, 2009‐2014. JAMA. 2017;318(13):1241–1249

Page 9: Evaluation and Management of Sepsis/Septic Shock in Adults ... › dam › AAFP › documents › events › fmx › hand… · Evaluation and Management of Sepsis/Septic Shock in

9

CDC Epidemiology JAMA 2017 multi-center study, 7,801,624 adult hospitalizations1:

– In 2014, 1.7 million Sepsis hospitalizations for adults in the US• 15.6% in-hospital mortality for sepsis, or 270,000 deaths• 6% discharged to hospice• “Sepsis was present in 35% of all hospitalizations” that led to death• 10% of all US deaths due to sepsis

– Adjusted annual Sepsis incidence remained stable, from 2009 - 2014• Based on Electronic Health Record (EHR)-based clinical data, not claims-based data

– 2009 - 2014, in-hospital mortality rates trended toward decline but lacked significance– “Discharge to hospice rates” did not change significantly

1. Rhee C, Dantes R, Epstein L, et al. Incidence and Trends of Sepsis in US Hospitals Using Clinical vs Claims Data, 2009‐2014. JAMA. 2017;318(13):1241–1249

Sepsis contributes to a large number of hospitalizations (35%) Sepsis causes 10% of deaths in the US annually There is not a large increase in the incidence of sepsis cases New reviews are needed with the new sepsis definition

Readmission Assessment1

• Sepsis - 20.4%

– $229 million/year

• Congestive Heart Failure - 23.6%

– $229 million/year

• Acute Myocardial infarction -17.7%

– $142 million/year

Drilling Down on Morbidity: Readmission Rates & Cost

1. Chang DW, Tseng C‐H, Shapiro MF. Rehospitalizations Following Sepsis: Common and Costly. Critical care medicine. 2015;43(10):2085‐2093.2. Chart from UPMC: Mayr F, Talisa V, Balakumar V. Proportion and Cost of Unplanned 30‐day readmissions after sepsis compared with other medical conditions. JAMA. 2017;317(5):530‐531

Page 10: Evaluation and Management of Sepsis/Septic Shock in Adults ... › dam › AAFP › documents › events › fmx › hand… · Evaluation and Management of Sepsis/Septic Shock in

10

CaseMs Jones, a 44-year old white woman with a history of hypertension, presents to clinic with abdominal pain that started within the last 2 days.

On further questioning, she endorses urinary frequency also.

Significant exam findings include temperature of 99.8 degrees Fahrenheit, respiratory rate of 18 breaths per minute, blood pressure of 119/68

Subtle Sepsis findings

You stop to think:Her blood pressure is

normally not well controlled: 155-160/89Respiratory rate?Temperature although

technically still normal is high

Page 11: Evaluation and Management of Sepsis/Septic Shock in Adults ... › dam › AAFP › documents › events › fmx › hand… · Evaluation and Management of Sepsis/Septic Shock in

11

From: Combatting Sepsis: A Public Health PerspectiveClin Infect Dis. Published online May 29, 2018. doi:10.1093/cid/ciy342Clin Infect Dis | Published by Oxford University Press for the Infectious Diseases Society of America 2018. This work is written by (a) US Government employee(s) and is in the public domain in the US.This work is written by (a) US Government employee(s) and is in the public domain in the US.

World Health Organization (WHO) 2017 Resolution1

Prevention– Vaccination– Infection prevention via clean water, hygiene education, emphasis on

clean childbirth, healthcare facility infection control

Increased clinician and public Sepsis awareness Early recognition by healthcare providers Early, rapid administration of antibiotics Emphasize judicious antibiotic administration Adherence to local sepsis guidelines Awareness and support for sepsis sequelae2

1. The World Health Organization Seventieth World Health Assembly Provision agenda item 12.1 “Improving the prevention, diagnosis and clinical management of sepsis” 13 April 2017. Accessed http://apps.who.int/gb/ebwha/pdf_files/WHA70/A70_13‐en.pdf?ua=1 2 September 2018 2.Reinhart K, Daniels R, Kissoon N, et al. Recognizing Sepsis as a Global Health Priority‐‐A WHO Resolution. N Engl J Med 2017; 377:414‐417Image taken from WHO website downloadable infographic. Accessed http://www.who.int/servicedeliverysafety/areas/sepsis/en/ 2 September 2018

Page 12: Evaluation and Management of Sepsis/Septic Shock in Adults ... › dam › AAFP › documents › events › fmx › hand… · Evaluation and Management of Sepsis/Septic Shock in

12

Improve Health Conditions

Educate Patients and Families aboutInfections, sepsis

THINK SEPSIS.ACT FAST

Reassess Patient Management

Poster curtesy of the Centers for Disease Control and Prevention.  8/2016

From the Clinic:

– Vaccination

– Comorbid disease control

– Close disease surveillance

– Education about sepsis

Sepsis in Clinic: PreventionPublic awareness still low1

• Numbers surveyed who have heard of sepsis

– 40% of Australians

– 55% in United States

– 62% in United Kingdom

– 69% of Germans

– 7% of Brazilians

1. Reinhart K, Daniels R, Kissoon N, et al. Recognizing Sepsis as a Global Health Priority‐‐A WHO Resolution. N Engl J Med 2017; 377:414‐417

Page 13: Evaluation and Management of Sepsis/Septic Shock in Adults ... › dam › AAFP › documents › events › fmx › hand… · Evaluation and Management of Sepsis/Septic Shock in

13

In Clinic: THINK SEPSIS• 70% of Sepsis

are community-acquired sepsis cases1

• Number 1 diagnosis is pneumonia2

1. Reinhart K, Daniels R, Kissoon N, et al. Recognizing Sepsis as a Global Health Priority‐‐A WHO Resolution. N Engl J Med 2017; 377:414‐4172. Novosad SA, Sapiano MR, Grigg C, et al. Vital Signs: Epidemiology of Sepsis: Prevalence of Health Care Factors and Opportunities for Prevention. MMWR Morb Mortal Wkly Rep 2016;65:864–869Poster curtesy of the Centers for Disease Control and Prevention.  8/2016

Sepsis in Clinic: ACT FAST

• Initial actions can include– Early antibiotic treatment

– Close monitoring for escalation of care

– Education of patient for warning signs

– Determine when hospitalization is warranted

1. Reinhart K, Daniels R, Kissoon N, et al. Recognizing Sepsis as a Global Health Priority‐‐A WHO Resolution. N Engl J Med 2017; 377:414‐417

Page 14: Evaluation and Management of Sepsis/Septic Shock in Adults ... › dam › AAFP › documents › events › fmx › hand… · Evaluation and Management of Sepsis/Septic Shock in

14

CDC 8/2016

Improve Health Conditions

Educate Patients and Families aboutInfections, sepsis

THINK SEPSIS.ACT FAST

Reassess Patient ManagementThe first 3 steps can be done in clinic!

Pathophysiology

“A systemic, dysregulated response to infection”1

Multi-faceted relationship between the inflammatory response and individual physiology is ‘non-linear’2,3,4

Genetics, comorbidities, life-style, age, and pathogen contribute3,4

Initial infection pro-inflammatory sepsis response “longer phase of relative immunosuppression.”5

1. Singer M, Deutschman C, Seymour C, et al. The Third International Consensus Definitions for Sepsis and Septic Shock (Sepsis‐3). JAMA 2016;315(8):801‐810.2.  Namas R, Zamora R, Namas R, An G et al. Sepsis: something Old, Something New, and a Systems View. J Crit Care. 2012; 27(3): 314.e1–314.e113. Van der Poll T. Future of Sepsis therapies. Crit Care. 2016; 20:1064. Gotts J, Matthay M. Sepsis: pathophysiology and clinical management. BMJ. 2016;353:i15855. Cohen J, Vincent JL, Adhikari N, et al. Sepsis: a roadmap for future research. Lancet Infectious Diseases Commission. Lancet. 2015; 15(5): 581‐614

Page 15: Evaluation and Management of Sepsis/Septic Shock in Adults ... › dam › AAFP › documents › events › fmx › hand… · Evaluation and Management of Sepsis/Septic Shock in

15

Pathophysiology

Illustration taken without alteration from Figure 1. Cavaillon JM, Eisen D, Annane D. Is boosting the immune system in sepsis appropriate. Crit Care. 2014;18(2):216Under Open Access license  attribution 4.0 International (CC by 4.0) Accessed https://creativecommons.org/licenses/by/4.0/ 

Pathophysiology: Big Picture

1. Systemic infection with immune dysregulation

2. Leads to endothelial and circulatory dysfunction

3. End Point: organ damage, metabolic derangement

1. Cohen J, Vincent JL, Adhikari N, Machado F, et al. Sepsis: a roadmap for future research. The Lancet Infectious Disease Commission. 2015; 15(5): P581‐6142. Gotts J, Matthay M. Sepsis: pathophysiology and clinical management. BMJ. 2016;353:i1585

Page 16: Evaluation and Management of Sepsis/Septic Shock in Adults ... › dam › AAFP › documents › events › fmx › hand… · Evaluation and Management of Sepsis/Septic Shock in

16

Microbiology

MDF Klebsiella pneumoniae

Thanks to the Center for Disease Control and National Institute of Allergy and Infectious Diseases. No alterations made

Staphylococcus aureus

• Respiratory Tract 35%1 / 63.5%2

• Urinary Tract 25%1 / 14.3%2

• Gastrointestinal 11%1 / 19.6%2

• Skin & Soft Tissue 11%1 / 6.6%2

• Bloodstream 5%1 / 15.1%2

• Bone & Joint 2%1

Microbiology by site

• Eye/Ear/Nose/Throat 1%1

• Central Nervous Systm <1%1 / 2.9%2

• Disseminated viral <1%1

• Surgical Site <1%1

• None documented 9%1

• Unknown/other 13%1 / 7.6%21. Novosad SA, Sapiano MR, Grigg C, et al. Vital Signs: Epidemiology of Sepsis: Prevalence of Health Care Factors and Opportunities for Prevention.  MMWR Morb Mortal Wkly Rep 2016;65:864–8692. Vincent JL, Rello J, Marshall J, et al. EPIC II Group of Investigators. International study of the prevalence and outcomes of infection in intensive care units. 

JAMA. 2009;302(21):2323‐9

CDC / EPIC IIInfection site

Infection site CDC / EPIC II

Page 17: Evaluation and Management of Sepsis/Septic Shock in Adults ... › dam › AAFP › documents › events › fmx › hand… · Evaluation and Management of Sepsis/Septic Shock in

17

Microbiology by pathogen

• Escherichia coli 12%1 / 16.0% • Streptococcus spp. 10.3%1 / 4.1%• Coagulase negative Staphylococcus spp. 9.7%1 / 10.8%• Staphylococcus aureus 9.0%1 / 20.5% • Pseudomonas spp. 2.07%1 / 19.9%• Bacillus spp. 2.07%1 / ---• Enterococcus spp. 2.07%1 / 10.9%• Enterobacter spp. 2.07%1 / 7.0%• Klebsiella spp. <1%1 / 12.7%• Acinetobacter spp. --- / 8.8%• Candida spp. <1%1 / 17.0%• Aspergillus --- / 1.4%• Anaerobes --- / 4.5%• Parasites --- / 0.7%

1. Novosad SA, Sapiano MR, Grigg C, et al. Vital Signs: Epidemiology of Sepsis: Prevalence of Health Care Factors and Opportunities for Prevention.  MMWR Morb Mortal Wkly Rep 2016;65:864–8692. Vincent JL, Rello J, Marshall J, et al. EPIC II Group of Investigators. International study of the prevalence and outcomes of infection in intensive care units. nJAMA. 2009;302(21):2323‐9

CDC / EPIC IIPathogen isolates

Microbiology

Methicillin‐resistantStaphylococcus aureus

Enterococcus sp.

Escherichia coli

Thanks to the Center for Disease Control and National Institute of Allergy and Infectious Diseases. No alterations made

Pseudomonas aeruginosa Candida sp.

Influenza virus

Page 18: Evaluation and Management of Sepsis/Septic Shock in Adults ... › dam › AAFP › documents › events › fmx › hand… · Evaluation and Management of Sepsis/Septic Shock in

18

🚧The ancient concept “σήψ𝝸ϛ” or sepsis– Etymology: Greek origin “𝜎𝜂𝜋𝜛,” “I rot”1

– First: Homer’s Iliad Epic1 in 7th-8th century BCE

– Referenced by Hippocrates, circa 460-365 BCE1,2,3

– A disease process of the body, aka putrefaction• Connotation: festering of wounds, rotting of flesh,

or smell of decay in a marsh3

– Contributed to Theory of Spontaneous Generation• Aristotle suggested ‘little creatures’ resulted from

the process of sepsis putrefaction3

Ancient Sepsis Origins

1. Geroulanos S, Douka E. Historical perspective of the word ”sepsis.” Intensive Care Med. 2006;32(12):2077 2. Botero JSH, Perez MCF. (2012). The History of Sepsis from Ancient Ecypt to the XIX Century. Sepsis 0 An Ongoing and Significant Challenge.  IntechOpen 2012. DOI: 10.5772/514843. Majno G. The ancient riddle of sigma eta psi iota sigma (sepsis). J Infect Dis 1991;163(5):937‐454.  Angus D, van der Poll T. Severe Sepsis and Septic Shock. N Engl J Med 2013;369:840‐51Illustration:  Public Domain, Ruebens sculpture of Hippocrates. https://commons.wikimedia.org/w/index.php?curid=1332072

Sepsis Diagnosis Development1

🚧 1928-1980s: Germ theory implementation in discovery and use of “anti-sepsis”1 therapies– “…new antibiotics and increasingly sophisticated critical care have had little impact on mortality rate”2

– Recognition that inflammatory and physiologic response to infection played a role in the disease1

🚧 1989: First diagnostic criteria by Dr. Roger Bone and Dr. Robert Balk from Chicago – Described sepsis syndrome as both infection and inflammatory response:

• Allow for “earlier detection…and possibly…earlier therapeutic intervention” • To “identify a population of patients at risk for the various complications of sepsis”2

– Defined as: “hypothermia…or hyperthermia…tachycardia, tachypnea, clinical evidence of an infection site” and “[A]t least one end-organ demonstrating inadequate perfusion or dysfunction”3

1. Namas R, Zamora R, Namaz R, An G, et al. Sepsis: Something Old, Something New, and a Systems View. J Crit Care. 2012;27(3):314.e1‐314.e112. American College of Chest Physicians/Society of Critical Care Medicine Consensus Conference: definitions for sepsis and organ failure, and guidelines for the use of innovative therapies in sepsis. Crit Care Med. 1992;20(6):864‐743. Bone R, Fisher C, Clemmer T, et al. Sepsis syndrome: a valid clinical entity. Crit Care Med. 1989; 17:389‐3934. Balk RA, Bone RC: The septic syndrome: definition and clinical implications. Crit Care Clin 1989;5:1‐8

Page 19: Evaluation and Management of Sepsis/Septic Shock in Adults ... › dam › AAFP › documents › events › fmx › hand… · Evaluation and Management of Sepsis/Septic Shock in

19

🚧 1991-1992: First consensus Sepsis definition1

– Developed by North American Consensus Conference

• American College of Chest Physicians, Society of Critical Care Medicine

• To centralize definitions, standardize diagnosis and research efficacy2

⚠ Definition of Systemic Inflammatory Response Syndrome (SIRS)⚠ SIRS present in both infections and non-

infectious inflammatory response disorders⚠ SIRS ≥ 2 of the following:

a) temperature >38℃ or <36℃b) heart rate > 90 beats/minute c) tachypnea > 20 breaths/minute or

hyperventilation of PaCO2 <32 torr d) white blood cell count > 12,000 cells/mm3 or

<4000 cells/mm3 or presence of >10% immature neutrophils

Society for Critical Care Medicine

⚠ “Sepsis” = SIRS + documented infection

⚠Severity based on organ dysfunction⚠Severe sepsis: sepsis + “hypotension

or hypoperfusion or organ dysfunction”1,2

⚠Septic shock: sepsis + hypotension, “despite adequate fluid resuscitation, along with…perfusion abnormalities”1

⚠Multiple organ dysfunction syndrome (MODS): “altered organ function in an acutely ill patient…hemostasis cannot be maintained without intervention”1

1. American College of Chest Physicians/Society of Critical Care Medicine Consensus Conference: definitions for sepsis and organ failure and guidelines for the use of innovative therapies in sepsis. Crit Care Med. 1992;20(6):864‐74

2. Bone R, Balk R, Cerra F et al. Definitions for sepsis and organ failure and guidelines for the use of innovative therapies in sepsis. The ACCP/SCCM Consensus Conference Committee. American College of Chest Physicians/Society of Critical Care Medicine. Chest. 1992;101(6):1644‐55

SIRS Era• Follow-up prospective and retrospective studies

followed– The natural clinical progression from SIRS to sepsis,

severe sepsis and septic shock1

• 35% SIRS developed Sepsis• 18% developed Severe Sepsis• 4% septic shock

– The correlation of SIRS with mortality1,2

• Increased with increase in # of positive SIRS criteria• Increased with number of days with positive SIRS2

1. Rangel‐Frausto MS, Pittet D, Costigan M, Hwang T, Davis CS, Wenzel RP. The natural history of the systemic inflammatory response syndrome (SIRS). A prospective study. JAMA. 1995;273(2):117‐232. Sun D, Aikawa N. The natural history of the systemic inflammatory response syndrome and the evaluation of SIRS criteria as a predictor of severity in patients hospitalized through emergency services. Keio J Med. 1999;48(1):28‐37

Page 20: Evaluation and Management of Sepsis/Septic Shock in Adults ... › dam › AAFP › documents › events › fmx › hand… · Evaluation and Management of Sepsis/Septic Shock in

20

🚧 2001: Sepsis Definition upgrade (Sepsis 2)– SCCM, European Society of Intensive

Care Medicine (ESICM), ACCP, American Thoracic Society (ATS) and Surgical Infection Society (SIS)

⚠ SIRS recognized as ‘overly sensitive and non-specific’1⚠ Expansion of signs and symptoms for

sepsis beyond SIRS1

– General parameters– Inflammatory parameters– Hemodynamic parameters– Organ dysfunction parameters– Tissue perfusion parameters

⚠ Focused on supporting ‘bedside’ diagnosis

Society for Critical Care Medicine

✅ Definition of Severe Sepsis unchanged

✅ Definition of Septic Shock unchanged for adults

1. Levy M, Fink M, Marshall J, et al. 2001 SCCM/ESICM/ACCP/ATS/SIS International Sepsis Definitions Conference. Intensive Care Med. 2003;29:530‐538

🚧 Treatment 2001: The Rivers Trial1

Prospective, single center study: emergency department septic patients

– Examined goal-directed therapy prior to intensive care unit (ICU) admission2,3,4

– Early goal-directed (EGDT) versus standard therapy

Results• In-hospital mortality 30.5% in EDGT group versus 46.5% in standard

therapy group (p=0.009)

• Between 7 and 72 hours– Lower lactate concentration in the EGDT group– Lower mean APACHE II score for the EGDT group

1. Rivers E, Nguyen B, Havstad S, et al. Early Goal‐Directed Therapy in the Treatment of Severe Sepsis and Septic Shock. N Engl J Med. 2001; 345:1368‐13772. Hayes MA, Timmins AC, Yau EH, et al. Elevation of systemic oxygen delivery in the treatment of critically ill patients. N Engl J Med. 1994;330(24):1717‐223. Martin C, Saux P, Eon B, Aknin P, Gouin F. Septic shock; a goal‐directed therapy using volume loading, dobutamine and/or norepinephrine. Acta Anaesthesiol Scand. 1990;34(5):413‐74. Shoemaker W, Appel P, Kram H. Role of oxygen debt in the development of organ failure sepsis and death in high‐risk surgical patients. Chest.1992;102(10:208‐15

Page 21: Evaluation and Management of Sepsis/Septic Shock in Adults ... › dam › AAFP › documents › events › fmx › hand… · Evaluation and Management of Sepsis/Septic Shock in

21

🚧 Treatment 2001: The Rivers Trial1

The Concepts:

⚠Sepsis ‘continuum’2 toward shock3

⚠Global hypoxia: poor oxygen delivery4,5 and increased oxygen demand ➡ mortality2

⚠EGDT controls cardiac function, balances oxygen delivery and demand1,3

⚠Maximizes “golden hours”1

• Regardless of hospital location• Focused on first 6 hour protocol

1. Rivers E, Nguyen B, Havstad S, et al. Early Goal‐Directed Therapy in the Treatment of Severe Sepsis and Septic Shock. N Engl J Med. 2001; 345:1368‐13772. Rangel‐Frausto MS, Pittet D, Costigan M, Hwang T, Davis CS, Wenzel RP. The natural history of the systemic inflammatory response syndrome (SIRS). A prospective study. JAMA. 1995;273(2):117‐233. Beal AL, Cerra F. Multiple organ failure syndrome in the 1990s: systemic inflammatory response and organ dysfunction. JAMA 1994;271:226‐2334. Hayes MA, Timmins AC, Yau EH, et al. Elevation of systemic oxygen delivery in the treatment of critically ill patients. N Engl J Med. 1994;330(24):1717‐225. Shoemaker W, Appel P, Kram H. Role of oxygen debt in the development of organ failure sepsis and death in high‐risk surgical patients. Chest.1992;102(10:208‐15

Adapted from Rivers E, et al. N Engl J Med. 2001

🚧 Treatment 2001: The Rivers Trial1

The Concepts:

Sepsis ‘continuum’2 toward shock3

Global hypoxia: poor oxygen delivery4,5 and increased oxygen demand mortality2

EGDT controls cardiac function, balances oxygen delivery and demand1,3

Maximizes “golden hours”1

• Regardless of hospital location• Focused on first 6 hour protocol

1. Rivers E, Nguyen B, Havstad S, et al. Early Goal‐Directed Therapy in the Treatment of Severe Sepsis and Septic Shock. N Engl J Med. 2001; 345:1368‐13772. Rangel‐Frausto MS, Pittet D, Costigan M, Hwang T, Davis CS, Wenzel RP. The natural history of the systemic inflammatory response syndrome (SIRS). A prospective study. JAMA. 1995;273(2):117‐233. Beal AL, Cerra F. Multiple organ failure syndrome in the 1990s: systemic inflammatory response and organ dysfunction. JAMA 1994;271:226‐2334. Hayes MA, Timmins AC, Yau EH, et al. Elevation of systemic oxygen delivery in the treatment of critically ill patients. N Engl J Med. 1994;330(24):1717‐225. Shoemaker W, Appel P, Kram H. Role of oxygen debt in the development of organ failure sepsis and death in high‐risk surgical patients. Chest.1992;102(10:208‐15

Adapted from Rivers E, et al. N Engl J Med. 2001

Early timing of treatment is importantConsolidated elements from prior studies into a sepsis protocolIf you follow the standard protocol, sepsis outcomes, including mortality, improve

Page 22: Evaluation and Management of Sepsis/Septic Shock in Adults ... › dam › AAFP › documents › events › fmx › hand… · Evaluation and Management of Sepsis/Septic Shock in

22

Founding year: 2002 Founding organizations:

• European Society of Intensive Care Medicine, Society of Critical Care Medicine, and International Sepsis Forum

Barcelona Declaration• For healthcare providers, governments• Group goal of 25% reduction in sepsis mortality• 5-point action plan for improving sepsis care1

Goal: influencing best practices to “reduce the burden of sepsis worldwide”2

• Focused on “severe sepsis or septic shock” care• No replacement for clinician bedside

“decision-making” capability”1

• Applicable in ICU and non-ICU settings

Surviving Sepsis Campaign 2003 SSC partners with Institute for Healthcare

Improvement (IHI)

2004-2005 Surviving Sepsis Campaign Care Bundles

2005 SSC publishes a manual for hospital use– 5000 copies disseminated in North America

1. “Action Demanded on World’s Oldest Killer: ‘Barcelona Declaration’ Aims to tackle 10th most common cause of death.” Surviving Sepsis Campaign. http://www.survivingsepsis.org/SiteCollectionDocuments/About‐Barcelona‐Declaration.pdf accessed 9/1/18 2. Dellinger R, Levy M, Rhodes A, Annane D, et al. Surviving Sepsis Campaign: international guidelines for management of severe sepsis and septic shock: 2012 Crit Care Med 2013; 41(2):580‐6373 Dellinger RP, Levy M, Carlet J, et al. Surviving Sepsis Campaign: International guidelines for management of severe sepsis and septic shock: 2008. Crit Care Med 2008;36(1):296‐327 

Surviving Sepsis Campaign🚧 First Guidelines in development 2003

⚠ 2004: “Surviving Sepsis Campaign guidelines for management of severe sepsis and septic shock”2

⚠ Focused on Severe Sepsis and Septic shock⚠ Implemented the initial 6 hour Rivers’ Early Goal Directed Therapy⚠ Delineated other expectations

⚠ Obtain blood cultures prior to antibiotics⚠ Broad spectrum, empiric antibiotics based on suspected pathogen

⚠ Reassess antibiotics after 48-72 hours to narrow⚠ Rapid source control – drain, debride, remove device, definitive control⚠ Fluids: crystalloid or colloid, boluses with reassessment⚠ Vasopressors – after fluid challenge. Preferred norepinephrine/dopamine⚠ Inotropes – dobutamine as needed.

⚠ No indication to elevate cardiac index unless the patient requires⚠ Steroids – helpful in patients in septic shock who require vasopressors

⚠ Consider Adrenocorticotropic hormone (ACTH) stimulation test⚠ Recombinant Human Activated Protein C (rhAPC) -- recommended in high APACHE II scores, septic shock, sepsis

with acute respiratory distress syndrome (ARDS) or multi-organ failure⚠ Blood product transfusions when hemoglobin is <7.0 g/dL⚠ ARDS ventilator management – low tidal volumes, recommended ARDSNET PEEP settings

1. “Action Demanded on World’s Oldest Killer: ‘Barcelona Declaration’ Aims to tackle 10th most common cause of death.” Surviving Sepsis Campaign. http://www.survivingsepsis.org/SiteCollectionDocuments/About‐Barcelona‐Declaration.pdf accessed 9/1/182.  Dillinger RP, Carlet JM, Masur H, et al. Surviving Sepsis Campaign guidelines for management of severe sepsis and septic shock. Crit Care Med. 2004;32(3):858‐73

Page 23: Evaluation and Management of Sepsis/Septic Shock in Adults ... › dam › AAFP › documents › events › fmx › hand… · Evaluation and Management of Sepsis/Septic Shock in

23

🚧 2008: Second edition of the guidelines published, focus on the SSC bundles2

Focused care into time-frame goals⚠ 6 hour:

⚠ lactate measurement, blood cultures, antibiotics ⚠ initial fluid administration by bolus, vasopressor

use if lactate > 4 mmol/L or persistent hypotension coupled with CVP and ScvO2monitoring

⚠ 24 hour (Management bundle): ✅ Consider steroids✅ rhAPC for severe sepsis and septic shock ✅ Glucose control✅ Mechanical ventilation control

Surviving Sepsis Campaign1

🚧 2012 SSC guidelines promoted new Bundle time intervals:3⚠ 3 Hour: serum lactate, blood cultures, appropriate

antibiotics ⚠ 30 mL/kilogram crystalloid bolus for hypotension

or lactate ≥ 4 mmol/L⚠ Removed colloid (hydroxyethyl starches)

recommendations3

⚠ Crystalloid fluids ideal. Albumin only in large fluid requirement situations

⚠ 6 Hour: vasopressors as needed to maintain MAP ≥ 65 mmHg, monitoring CVP, SCVO2

⚠ Recommendations changed: norepinephrine epinephrine vasopressin.

⚠ Dopamine considered alternate option⚠ No rhAPC⚠ Avoid hydrocortisone in septic shock except

when requiring maximal therapy and unable to maintain hemodynamic stability3

1. Timeline adapted from the Surviving Sepsis Campaign History webpages http://www.survivingsepsis.org/About‐SSC/Pages/History.aspx. Accessed 9.1.18.2. Dellinger RP, Levy M, Carlet J, et al. Surviving Sepsis Campaign: International guidelines for management of severe sepsis and septic shock: 2008. Crit Care Med 2008;36(1):296‐3273. Dellenger RP, Levy M, Rhodes A, et al. Surviving Sepsis Campaign: International Guidelines for Management of Severe Sepsis and Septic Shock: 2012. Crit Care Med. 2013;41(2):580‐6374.  based on VISEP, CRYSTMAS, 6S, CHST trials, also the CRYSTAL trial supported but not included. 

“Rory’s Regulations” - 2013✳ New York State Department of Health mandated-adherence to sepsis protocols

New York Codes, Rules, and Regulations parts 405.2 and 405.41

The New York State Law mandates Hospitals follow SSC Bundles for 3 and 6 hours1

• Treatment mandated based on SSC bundles, using Sepsis 2 definitions• Individual physician diagnostic method not mandated• Hospitals must update their ‘early recognition’ protocols based on evolving evidence

Early results:• Improved protocol adherence (2014: 73.7% - 2016: 84.3%)2

• Mortality: 30.2% in 2014 to 26.0% in 2016 – no prior trends to compare2,3

✳ Illinois became second state in 2016, New Jersey in 2018.• States considering mandates include Pennsylvania, Ohio, California, Wisconsin3

The Rory Staunton Foundation goal implementation of protocols in all 50 states4

• Ongoing debate in the medical community about state-mandates for medical care

1.New York Codes, Rules and Regulations, parts 405.2 and 405.4. Accessed https://www.health.ny.gov/facilities/public_health_and_health_planning_council/meetings/2013‐02‐07/docs/13‐01.pdf 2 September 20182. New York State Department of Health. New York State report on sepsis care improvement initiative: hospital quality performance. March 2018 Accessed https://www.health.ny.gov/press/reports/docs/2016_sepsis_care_improvement_initiative.pdf 2 September 20183. Hershey T, Kahn J. State Sepsis Mandates –A new era for regulation of Hospital quality. N Engl J Med. 2017;376:2311‐2313 4. Centers for Disease Control and Prevention. “The Power of Families in the Battle against Sepsis” by Orlaith Staunton. Accessed https://blogs.cdc.gov/safehealthcare/the‐power‐of‐families‐in‐the‐battle‐against‐sepsis/ 2 September 2018

Page 24: Evaluation and Management of Sepsis/Septic Shock in Adults ... › dam › AAFP › documents › events › fmx › hand… · Evaluation and Management of Sepsis/Septic Shock in

24

Standard therapy versus EGDT• ProCESS trial1

– 31 emergency departments, 1341 patients in United States– Randomized patients in septic shock to either protocol-based EGDT, protocol-based standard therapy, or usual care

• Protocol-based standard therapy: no central venous catheter, inotropes or blood transfusions required, only as needed• Usual Care: ”bedside providers” directed care

– Findings: protocol based resuscitation for septic shock did not lead to better mortality outcome at 60 days, 90 days, 1-year, or affect the need for organ support1 (90-day mortality for protocol-based EGDT 21.0% versus protocol based standard therapy 18.2% versus usual care 18.9%)

• ARISE2

– 51 emergency departments, 1600 patients in Australia, New Zealand– Randomization of patients in early septic shock to EGDT or ’usual care’ for 6 hours for resuscitation

• Usual Care: the hospital protocols did not have sepsis care guided by protocol nor was measurement of ScvO2 utilized– Findings: more patients in EGDT vs usual care groups received vasopressor infusion (66..6% vs 57.8%), dobutamine (15.4% vs 2.6%), red-cell

transfusion (13.6% vs 7.0%) (P<0.001)– MAP was similar at end of the 6 hours with EGDT 76.5 ± 10.8 mm Hg vs usual care 75.3 ± 11.4 mm Hg (p=0.04)– Primary outcome of death all-cause mortality at 90-days did not differ (EGDT 18.6% vs usual care 18.8%) (p =0.90)

• 28-day mortality and in-hospital mortality, length of hospital stay all non-significant differences

• ProMISe3

– 56 hospitals, 1260 patients in England– Randomization Emergency room patients to EGDT or ‘usual care’– Primary Outcome of all-cause mortality at 90 days: no difference (EGDT 29.5% versus usual care 29.5%), absolute risk reduction -0.3%– Findings of EGDT associated with higher mean SOFA score at 6 hours (EGDT 6.4 versus usual care 5.6, p<0.001), more days on advanced

cardiovascular support, longer ICU stays

1. The ProCESS Investigators. A Randomized Trial of Protocol‐Based Care for Early Septic Shock. N Engl J Med . 2014;370:1683‐16932. The ARISE Investigators and the ANZICS Clinical Trials Group. Goal‐Directed Resuscitation for Patients with Early Septic Shock. N Engl J Med 2014; 371:1496‐15063. Mouncey P, Osborn T, Power P, et al. Trial of Early, Goal‐Directed Resuscitation for Septic Shock. N Engl J Med . 2015;372:1301‐1311

Standard therapy versus EGDT✳ Intensive Care Medicine 2015

– Systematic review of 5 randomized clinical trials including ARISE, ProCESS, ProMISe– Results: Early Goal Directed Therapy had ‘no effect’ on mortality vs control group

(pooled OR 1.01). EGDT “not superior to usual care”1

✳New England Journal 2017, PRISM study– Meta-analysis of individual patient data from ARISE, ProCESS, ProMISE

• 3723 patients, 138 hospitals, 7 countries– Outcome the same: 90-day mortality for EGDT 24.9% vs usual care 25.4%. OR 0.97 EGDT group had significantly longer ICU care than usual care No benefit from EGDT even in higher risk shock patients No 90-day mortality benefit

1. Angus DC, Barnato AE, Bell D, et al. A systematic review and meta‐analysis of early goal‐directed therapy for septic shock: the ARISE, ProCESS and ProMISe Investigators. Intensive Care Med. 2015;41(9):1549‐602. PRISM Investigators Rowan KM, Angus DC, Bailey M, et al. Early, Goal‐Directed Therapy for Septic Shock –A patient level meta‐analysis. N Engl J Med. 2017; 376(23):2223‐2234

Page 25: Evaluation and Management of Sepsis/Septic Shock in Adults ... › dam › AAFP › documents › events › fmx › hand… · Evaluation and Management of Sepsis/Septic Shock in

25

Standard therapy versus EGDT• Intensive Care Medicine 2015

– Systematic review of 5 randomized clinical trials including ARISE, ProCESS, ProMISe– Results: Early Goal Directed Therapy had ‘no effect’ on mortality vs control group

(pooled OR 1.01). EGDT “not superior to usual care”1

• New England Journal 2017, PRISM study– Meta-analysis of individual patient data from ARISE, ProCESS, ProMISE

• 3723 patients, 138 hospitals, 7 countries– Outcome the same: 90-day mortality for EGDT 24.9% vs usual care 25.4%. OR 0.97 EGDT group had significantly longer ICU care No benefit from EGDT even in higher risk shock patients No 90-day mortality benefit

1. Angus DC, Barnato AE, Bell D, et al. A systematic review and meta‐analysis of early goal‐directed therapy for septic shock: the ARISE, ProCESS and ProMISe Investigators. Intensive Care Med. 2015;41(9):1549‐602. PRISM Investigators Rowan KM, Angus DC, Bailey M, et al. Early, Goal‐Directed Therapy for Septic Shock –A patient level meta‐analysis. N Engl J Med. 2017; 376(23):2223‐2234

Consistent with individual trial findings:No difference in mortality

The question remains…Did the implementation of EGDT management and the SSC bundles influence ‘usual care?’

🚧 2016 Sepsis Concept changes (Sepsis 3):

🛑 New Sepsis Definition:⚠ ”Life-threatening organ dysfunction due to

a dysregulated host response to infection.”1

⚠ Defined by Sequential Organ Failure Assessment (SOFA) criteria ⚠Quick SOFA (qSOFA) criteria

❌ Abandoned SIRS-based definition

⚠ Distinguishing SIRS prediction of physiologic response to infection from dysregulation tied to increased mortality and morbidity2

⚠Sequential Organ Failure Assessment Score of ≥ 2 points, associated with >10% in-hospital mortality1

Society for Critical Care Medicine

❌ Term no longer used: ‘severe sepsis’

❌ Term no longer used: ‘septicemia’

⚠ Septic Shock clarification: ⚠ Hypotension requiring vasopressors

to maintain mean blood pressure ≥ 65 mmHg + persistent serum lactate >2 mmol/L after adequate fluid resuscitation3

☠ Associated in-hospital mortality > 40%1

1. Singer M, Deutschman C, Seymour C, et al. The Third International Consensus Definitions for Sepsis and Septic Shock (Sepsis‐3). JAMA 2016;315(8):801‐810.2. Seymour C, Liu V, Iwashyna T, et al. Assessment of Clinical Criteria for Sepsis: For the Third International Consensus Definitions for Sepsis and Sept ic Shock (Sepsis‐3). JAMA 2016;315(8):762‐7743.  Shakar‐Hari M, Phillips G, Levy M, et al. Developing a New Definition and Assessing New Clinical Criteria for Septic Shock: For the Third International Consensus Definitions for Sepsis and Septic Shock (Sepsis – 3). JAMA 2016; 315(8):775‐787

Page 26: Evaluation and Management of Sepsis/Septic Shock in Adults ... › dam › AAFP › documents › events › fmx › hand… · Evaluation and Management of Sepsis/Septic Shock in

26

🚧 2016 Sepsis Concept changes (Sepsis 3):

🛑 New Sepsis Definition:⚠ ”Life-threatening organ dysfunction due to

a dysregulated host response to infection.”1

⚠ Defined by Sequential Organ Failure Assessment (SOFA) criteria ⚠Quick SOFA (qSOFA) criteria

❌ Abandoned SIRS-based definition

⚠ Distinguishing SIRS prediction of physiologic response to infection from dysregulation tied to increased mortality and morbidity2

⚠Sequential Organ Failure Assessment Score of ≥ 2 points, associated with >10% in-hospital mortality1

Society for Critical Care Medicine

❌ Term no longer used: ‘severe sepsis’

❌ Term no longer used: ‘septicemia’

⚠ Septic Shock clarification: ⚠ Hypotension requiring vasopressors

to maintain mean blood pressure ≥ 65 mmHg + persistent serum lactate >2 mmol/L after adequate fluid resuscitation3

☠ Associated in-hospital mortality > 40%1

1. Singer M, Deutschman C, Seymour C, et al. The Third International Consensus Definitions for Sepsis and Septic Shock (Sepsis‐3). JAMA 2016;315(8):801‐810.2. Seymour C, Liu V, Iwashyna T, et al. Assessment of Clinical Criteria for Sepsis: For the Third International Consensus Definitions for Sepsis and Sept ic Shock (Sepsis‐3). JAMA 2016;315(8):762‐7743.  Shakar‐Hari M, Phillips G, Levy M, et al. Developing a New Definition and Assessing New Clinical Criteria for Septic Shock: For the Third International Consensus Definitions for Sepsis and Septic Shock (Sepsis – 3). JAMA 2016; 315(8):775‐787

Everything Changes!

Society for Critical Care Medicine✳2016: Sepsis Concept changes (Sepsis 3)✳ New Sepsis Definition: How to diagnose Sepsis

✳ ICU patients: SOFA score ≥ 2

✳Emergency Department/non-ICU hospitalized patients:⬇Suspected infection – calculate qSOFA to determine mortality risk

⬇qSOFA positive for ≥2 of 3 criteria: Respiratory rate ≥ 22 breaths per minute

Glasgow Coma Scale (GCS) < 15

Systolic Blood pressure ≤100 mmHg1,2

1. Singer M, Deutschman C, Seymour C, et al. The Third International Consensus Definitions for Sepsis and Septic Shock (Sepsis‐3). JAMA 2016;315(8):801‐810.2. Seymour C, Liu V, Iwashyna T, et al. Assessment of Clinical Criteria for Sepsis: For the Third International Consensus Definitions for Sepsis and Sept ic Shock (Sepsis‐3). JAMA 2016;315(8):762‐7743. Shakar‐Hari M, Phillips G, Levy M, et al. Developing a New Definition and Assessing New Clinical Criteria for Septic Shock: For the Third International Consensus Definitions for Sepsis and Septic Shock (Sepsis –3). JAMA 2016; 315(8):775‐787

Page 27: Evaluation and Management of Sepsis/Septic Shock in Adults ... › dam › AAFP › documents › events › fmx › hand… · Evaluation and Management of Sepsis/Septic Shock in

27

SOFA score 0 1 2 3 4

Respiratory systemPaO2/FiO2

mmgHg

≥400  <400 <300 <200 + respiratory support

<100 + respiratory support

Hepatic systemBilirubin (mg/dL) <1.2 1.2‐1.9 2.0‐5.9 6.0‐11.9 >12.0

Cardiovascular systemMAP (mean arterial pressure)

MAP ≥ 70 mmHg

MAP<70 mmHg

Dopamine <5 𝛍g/kg/min or 

dobutamine any dose

Dopamine 5.1‐15 or epinephrine ≤0.1 or norepinephrine 

≤0.1

Dopamine > 15 or epinephrine > 0.1 or norepinephrine > 

0.1

Renal SystemCreatinine (mg/dL)Urine Output (mL/d)

<1.2‐

1.2‐1.9‐

2.0‐3.4‐

3.5‐4.9<500

>5.0<200

Central Nervous SystemGlasgow coma scale 15 13‐14 10‐12 6‐9 <6

CoagulationPlatelets, x 103/𝛍L ≥150 <150 <100 <50 <20

Adapted from Sequential Organ Failure Assessment Score, Singer M, Deutschman C, Seymour C, et al. The Third International Consensus Definitions for Sepsis and Septic Shock (Sepsis‐3). JAMA 2016;315(8):801‐810. and Vincent JL, de Mendonca A, Cantraine F, et al. Use of the SOFA score to assess the incidence of organ dysfunction/failure in intensive care units: results of a multicenter prospective study. Working Group on “Sepsis‐Related Problems” of the  European Society of Intensive Care Medicine. Crit Care Med. 1998; 26 (11): 1793‐1800

SOFA score 0 1 2 3 4

Respiratory systemPaO2/FiO2

mmgHg

≥400  <400 <300 <200 + respiratory support

<100 + respiratory support

Hepatic systemBilirubin (mg/dL) <1.2 1.2‐1.9 2.0‐5.9 6.0‐11.9 >12.0

Cardiovascular systemMAP (mean arterial pressure)

MAP ≥ 70 mmHg

MAP<70 mmHg

Dopamine <5 𝛍g/kg/min or 

dobutamine any dose

Dopamine 5.1‐15 or epinephrine ≤0.1 or norepinephrine 

≤0.1

Dopamine > 15 or epinephrine > 0.1 or norepinephrine > 

0.1

Renal SystemCreatinine (mg/dL)Urine Output (mL/d)

<1.2‐

1.2‐1.9‐

2.0‐3.4‐

3.5‐4.9<500

>5.0<200

Central Nervous SystemGlasgow coma scale 15 13‐14 10‐12 6‐9 <6

CoagulationPlatelets, x 103/𝛍L ≥150 <150 <100 <50 <20

Adapted from Sequential Organ Failure Assessment Score, Singer M, Deutschman C, Seymour C, et al. The Third International Consensus Definitions for Sepsis and Septic Shock (Sepsis‐3). JAMA 2016;315(8):801‐810. and Vincent JL, de Mendonca A, Cantraine F, et al. Use of the SOFA score to assess the incidence of organ dysfunction/failure in intensive care units: results of a multicenter prospective study. Working Group on “Sepsis‐Related Problems” of the  European Society of Intensive Care Medicine. Crit Care Med. 1998; 26 (11): 1793‐1800

SOFA Score:An increase of ≥ 2 points = sepsis

Page 28: Evaluation and Management of Sepsis/Septic Shock in Adults ... › dam › AAFP › documents › events › fmx › hand… · Evaluation and Management of Sepsis/Septic Shock in

28

SIRS criteria includes most infections, not just the severe ones

qSOFA identifies risk for mortality– Infectious patients (i.e. sepsis)

– Non-infectious, acutely ill patients

qSOFA ≠ the new SIRS criteria1

1. Vincent JL, Martin G, Levy M. qSOFA does not replace SIRS in the definition of sepsis. Crit Care. 2016; 20:210Illustration  reproduced without alternation  from  Vincent JL et al, Crit Care 2016. under Creative Commons 

Attribution 4.0 International license (http://creativecommons.org/licenses/by/4.0/)

Society for Critical Care Medicine 2016: Sepsis Concept changes (Sepsis 3)

The argument against the SIRS criteria Cases of ‘SIRS-negative severe sepsis’1 identified 2015 NEJM study out of the ANZICS group, retrospective review of SIRS correlation with infection and organ failure,

co-utilizing the APACHE II and III scores. 1,171,797 patients, 172 ICUs In the ICU setting, “Of the patients with infection and organ dysfunction, 96,385 (87.9%) had SIRS-positive severe sepsis

and 13,278 (12.1%) had SIRS-negative severe sepsis.” 20% met no SIRS criteria and 80% met one SIRS criterion. Findings: “the SIRS-criteria rule missed one patient in eight with severe sepsis”1

2006 Intensive Care Medicine multicenter, cohort observational study involving all new ICU admissions (n=3,147) found that 93% met ≥ 2 SIRS criteria2

This finding was similar to other studies in the 1990s and 2000s

For ICU patients (and possibly non-ICU also), SIRS holds poor predictive value for the infected patient who is at increased mortality risk, and is not specific enough to distinguish between ”sterile tissue injury” and sepsis with infection3

Further, by definition, the SIRS criteria will be present in most patients with infection –whether life threatening or not4

1. Kaukonen KM, Bailey M, Pilcher D, Cooper DJ, Bellomo R. Systemic inflammatory response syndrome criteria in defining severe sepsis. N Engl J Med 2015;372(17):1629‐38.2. Sprung CL, Sakr Y, Vincent JL, et al. An evaluation of systemic inflammatory response syndrome signs in the Sepsis occurrence in Acutely Ill Patients (SOAP) study. Intensive Care Med. 2006 32(3):421‐73. Vincent JL, Opal S, Marshall J, Tracey K. Sepsis definitions: time for change. Lancet. 2013;381(9868):774‐7754. Vincent JL, Martin G, Levy M. qSOfA does not replace SIRS in the definition of sepsis. Crit Care. 2016; 20:210

Page 29: Evaluation and Management of Sepsis/Septic Shock in Adults ... › dam › AAFP › documents › events › fmx › hand… · Evaluation and Management of Sepsis/Septic Shock in

29

Society for Critical Care Medicine✳2016: Sepsis Concept changes (Sepsis 3) Validation for SOFA, qSOFA

JAMA 2017 French study prospectively compared qSOFA to SIRS and Severe Sepsis definitions in 30 emergency departments, with follow up to discharge or death. qSOFA correlated better with overall in-hospital mortality (AUROC 0.80) versus SIRS/Severe Sepsis (AUROC 0.65). Hazard ratio for death: qSOFA 6.2 vs 3.5 for severe sepsis1

European Society of Clinical Microbiology and Infectious Diseases 2017 Greek study; prospective cohort 3346 patients outside of the ICU, 1058 ICU patients. SIRS versus qSOFA Mortality for SIRS 25.3% and qSOFA 41.2% (p<0.0001). Neither very predictive SOFA sensitivity for 28 day mortality: outside the ICU 87.5% and inside the ICU 95.8%2

JAMA 2017 ANZICS retrospective study compared SOFA, SIRS and qSOFA scoring for ICU patients in Australia and New Zealand ICUs, within 24 hours of admission.

SOFA scoring discriminated in-hospital mortality: AUROC 0.753 vs SIRS criteria AUROC 0.589 vs qSOFA AUROC 0.6073

JAMA 2018 SAILORS retrospective study in 10 low and middle income countries qSOFA a stronger mortality predictor (low – high score increase of 13%, 95% CI 11-14%) vs SIRS (low—high score increase of 5%, 95% CI 3-6%)4

Several systematic literature review studies have identified poor sensitivity (AUC 0.47)5 - for qSOFA outside the ICU for ‘adverse outcomes’ 5,6

but high specificity for mortality

1. Freund Y, Lemachatti N, Krastinova E, et al. Prognostic Accuracy of Sepsis‐e Criteria for In‐Hospital Mortality among patients with suspected infection presenting to the emergency department. JAMA. 2017;317(3):301‐3082. Giamarellos‐Bourboulis EJ, Tsaganos T, Tsangaris I, et al. Validation of the new Sepsis‐3 definitions: proposal for improvement in early risk identification. Clin Microbiol Infect. 2017;23(2):104‐1093. Raith EP, Udy AA, Bailey m, et al; Australian and New Zealand Intensive Care Society (ANZICS) Center for Outcomes and Resource Evaluation (CORE). Prognostic Accuracy of the SOFA Score, SIRS Criteria, and qSOFA score for in‐hospital mortality among adults with suspected

infection admitted to the intensive care unit. JAMA. 2017;317(3):290‐3004. Rudd K, Seymour C, Aluisio A, et al. Association of the Quick Sequential (Sepsis‐Related) Organ Failure Assiessment (qSOFA) Score with Excess Hospital Mortality in Adults with Suspected Infection in Low and Middle‐Income Countries. JAMA 2018; 319(21):2202‐2211.5. Song JU, Sin CK, Park HK, Shim SR, Lee J. Performance of the quick Sequential (sepsis‐related) Organ Failure Assessment score as a prognostic tool in infected patients outside the intensive care unit: a systematic review and meta‐analysis. Crit Care. 2018;22(1):286. Maitra S, Som A, Bhattacharjee S. Accuracy of quick Sequential Organ Failure Assessment (qSOFA) score and systemic inflammatory response syndrome (SIRS) criteria for predicting mortality in hospitalized patients with suspected infection: a meta‐analysis of observational studies. Clin Microbiol Infect. 2018; pii: S1198‐743X(19)30294‐5

Society for Critical Care Medicine✳2016: Sepsis Concept changes (Sepsis 3)

✳Meta-analysis: SIRS vs qSOFA Annals of Internal Medicine 2018 Canadian meta-analysis, 38 studies

(n=385,333) ICU populations qSOFA sensitivity higher (87.2%) than non-ICU populations (51.2%)

ED patient population: qSOFA pooled sensitivity and specificity (46.7% and 81.3%) vs SIRS pooled sensitivity and specificity (83.6% and 30.6%)

In non-ICU patients, likelihood ratios (LR): qSOFA positive LR 2.50 and negative LR 0.61; SIRS positive LR 1.25 and negative LR 0.52

Short-term mortality: qSOFA pooled sensitivity 60.8%, pooled specificity 72.0%, SIRS criteria pooled sensitivity 88.1%, specificity 25.8%

1. Fernando S, Tran A, Taljaard M, et al. Prognostic Accuracy of the Quick Sequential Organ Failure Assessment for Mortality in patients with suspected infection. Ann Intern Med. 2018;168:266‐275. 

Page 30: Evaluation and Management of Sepsis/Septic Shock in Adults ... › dam › AAFP › documents › events › fmx › hand… · Evaluation and Management of Sepsis/Septic Shock in

30

Practical application: EHR sepsis alerts

• Currently many hospitals utilize SIRS-based alerts for sepsis and acutely-ill patient identification

• EHR Alert “fires” when predetermined criteria are “met”

Critical Care Medicine 2017 study SIRS criteria more sensitive for non-ICU patients

(91%) compared to 54% for qSOFA, 70% for MEWS and 67% for NEWS

SIRS criteria ‘fired’ earliest, 12 hours before qSOFA1

1. Churpek M, Snyder A, Han X, et al. Quick Sepsis‐related Organ Failure Assessment, Systemic Inflammatory Response Syndrome, and Early Warning Scores for Detecting ClinicalDeterioration in Infected patients outside the Intensive Care Unit. Am J Respir Crit Care Med. 2017;195(7):906‐911

The debate continues

What we know about sepsis now:1 SOFA predicts ICU mortality best

2 qSOFA seems to predict in-hospital mortality better than SIRS

but the jury is still out…

3 SIRS criteria is still the more sensitive screening test for

identifying early sepsis and increased risk of mortality1

4 None of the tests are ideal – need for a Sepsis 4?

1.Fernando S, Tran A, Taljaard M, et al. Prognostic Accuracy of the Quick Sequential Organ Failure Assessment for Mortality in patients with suspected infection. Ann Intern Med. 2018;168:266‐275.

Page 31: Evaluation and Management of Sepsis/Septic Shock in Adults ... › dam › AAFP › documents › events › fmx › hand… · Evaluation and Management of Sepsis/Septic Shock in

31

The debate continues

Clinical application:

✳ When admitting a patient you suspect to have

infection to a non-ICU hospital bed, calculate

SIRS, if positive, then calculate qSOFA1,2

✳ qSOFA can guide disposition of the patient to

the correct level of care based on mortality risk1,2

1..”Surviving Sepsis Campaign Responds to Sepsis‐3” March 1, 2016. http://www.survivingsepsis.org/SiteCollectionDocuments/SSC‐Statements‐Sepsis‐Definitions‐3‐2016.pdf. Accessed 9/1/182.Bhattacharjee P, Edelson D, Churpek M. Identifying patients with sepsis on the hospital wards. Chest. 2017; 151(4):898‐907

AES Question 3A 67 year-old man with a history of poorly-controlled hypertension, diabetes mellitus type 2, and tobacco use comes in during your walk-in clinic hours, concerned that his blood glucose at home was ‘320’ earlier today. On further review, he reports chills and a cough that started today. Exam reveals a temperature of 99.1℉, Heart Rate 111 beats/minute, Blood pressure 121/58, respiratory rate of 25 breaths/minute, Point of Care glucose of 310. You observe a tired-appearing African American man, clear-sounding lungs but tachypneic, speaking in 3-4 word phrases. Based on the current sepsis definition, what is the most concerning sign for sepsis in this patient?

A. Clinical appearanceB. Heart rateC. TemperatureD. Blood pressureE. Respiratory rateF. Serum glucose

Page 32: Evaluation and Management of Sepsis/Septic Shock in Adults ... › dam › AAFP › documents › events › fmx › hand… · Evaluation and Management of Sepsis/Septic Shock in

32

Question 3A 67 year-old man with a history of poorly-controlled hypertension, diabetes mellitus type 2, and tobacco use comes in during your walk-in clinic hours, concerned that his blood glucose at home was ‘320’ earlier today. On further review, he reports chills and a cough that started today. Exam reveals a temperature of 99.1℉, Heart Rate 111 beats/minute, Blood pressure 121/58, respiratory rate of 25 breaths/minute, Point of Care glucose of 310. You observe a tired-appearing African American man, clear-sounding lungs but tachypneic, speaking in 3-4 word phrases. Based on the current sepsis definition, what is the most concerning sign for sepsis in this patient?

A. Clinical appearanceB. Heart rateC. TemperatureD. Blood pressureE. Respiratory rateF. Serum glucose

Surviving Sepsis Campaign

✳ 2016 SSC guideline Highlights⚠Sepsis and Septic Shock = medical emergencies

⚠Begin treatment immediately (<1 hour)

✅If hypoperfusion: ≥ 30 mL/kilogram IV crystalloid in first 3 hours

✅Resuscitation guided by lactate, goal to normalize

⚠Dynamic reassessment for fluid responsiveness emphasized

✅MAP goal ≥ 65 mm Hg

1. Rhodes A, Evans L, Alhazzani W. Surviving Sepsis Campaign: International Guielines for Management of Sepsis and Septic Shock: 2016. Crit Care Med. 2017;45(3);486‐552

Page 33: Evaluation and Management of Sepsis/Septic Shock in Adults ... › dam › AAFP › documents › events › fmx › hand… · Evaluation and Management of Sepsis/Septic Shock in

33

IDSAThe Infectious Disease Society of America (IDSA) withheld endorsement from the Surviving Sepsis Campaign’s 2016 guidelines1

Concerns raised about the guidelines:

1. The guidelines’ ”failure to acknowledge the practical difficulties” for clinicians in diagnosing sepsis– Need to determine balance benefit vs harm in treating patients empirically

2. The guidelines do not differentiate well between “suspected sepsis and suspected septic shock”– Multiplicity of definitions– Risk for “one-size-fits-all” treatment: antibiotic and fluid resuscitation overuse

3. The guidelines ‘overly rigid’ to be used by hospitals and health plans for quality measures– Unintended consequences: administration of broad spectrum antibiotics to uninfected patients– Difficulty to agree upon time points for treatment– Challenge of determining feasibility for rapid antibiotic administration

I..DSA Sepsis Task Force; Infectious Diseases Society of America (IDSA) POSITION STATEMENT: Why IDSA Did Not Endorse the Surviving Sepsis Campaign Guidelines, Clinical Infectious Diseases, Volume 66, Issue 10, 2 May 2018, Pages 1631–1635

Surviving Sepsis Campaign 20181

New⚠ Hour-1 Bundle: combined 3-hour and 6-hour treatment bundles in the first hour of

diagnosis⚠ Remeasure 2-4 hours if lactate is > 2 mmol/L⚠ Start vasopressors if the patient “hypotensive during or after” receiving fluids (< 6 hours)– ”Time zero” – in the Emergency Department or elsewhere once all criteria for Sepsis or

Septic Shock are met

Physician response:• Concerns that accelerated time frame impairs physician ability to assess for sepsis –

petition to abandon SSC guidelines2

• Concerns have arisen regarding the impartiality of the Surviving Sepsis Campaign3

• IDSA does not support the SSC guidelines4

1. Levy, M.M., Evans, L.E. & Rhodes, A. Intensive Care Med (2018) 44: 9252. Petition to retire the surviving sepsis campaign guidelines. Josh Farkas May 2, 2018. https://emcrit.org/pulmcrit/ssc‐petition. Accessed 7/14/183. Eichacker P, Natanson C, Danner R. Surviving Sepsis—Practice Guidelines, Marketing Campaigns, and Eli Lilly. N Engl J Med 2006; 355:1640‐16424.. IDSA Sepsis Task Force; Infectious Diseases Society of America (IDSA) POSITION STATEMENT:Why IDSA Did Not Endorse the Surviving Sepsis Campaign Guidelines, Clinical Infectious Diseases, Volume 66, Issue 10, 2 May 2018, Pages 1631–1635

Page 34: Evaluation and Management of Sepsis/Septic Shock in Adults ... › dam › AAFP › documents › events › fmx › hand… · Evaluation and Management of Sepsis/Septic Shock in

34

Sep-1: CMS/Medicare✳The Centers for Medicare & Medicaid Services (CMS) incorporated the SSC

Bundles into the Severe Sepsis and Septic Shock Early Management Bundle (Sep-1)

✳Sep-1: measure for Hospital Inpatient Quality Reporting Program (IQR) instituted by CMS

✳Effective 2015, individual hospital compliance data for the 3- and 6-hour Sepsis (SSC 2012) bundles must be reported to CMS

✳This data is made available to the public: goal to improve quality✳Not tied to CMS reimbursement regardless of individual hospital compliance rates1

• CMS reimbursement is only tied to a failure to report data1,2

1. Townsend S, Tefera L, Rivers E. Evidence Underpinning the Centers for Medicare & Medicaid Services’ severe sepsis and septic shock management bundle (SEP‐1). Ann Intern Med. 2018;168(8):609‐6102. Centers for Medicare & Medicaid Services. Hospital Inpatient Quality Reporting Program. Updated 9 September 2017.Accessed at https://www.cms.gov/Medicare/Quality‐Initiatives‐Patient‐Assessment‐Instruments/HospitalQualityInits/HospitalRHQDAPU.html on 2 September 2018Logo accessed without alteration. https://commons.wikimedia.org/wiki/File:Centers_for_Medicare_and_Medicaid_Services_logo_2014.png 

Sep-1: CMS & JCAHO

✳ The Sep-1 CMS Sepsis bundle is jointly identified by The Joint Commission on Accreditation of Health Care Organizations (JCAHO)1

1. The Joint Commission. Specifications manual for national hospital inpatient quality measures. 2018. Accessed at https://www.jointcommission.org/specifications_manual_for_national_hospital_inpatient_quality_measures.aspx on 2 September 2018

Page 35: Evaluation and Management of Sepsis/Septic Shock in Adults ... › dam › AAFP › documents › events › fmx › hand… · Evaluation and Management of Sepsis/Septic Shock in

35

Sep-1: Sepsis BundlesDiagnosis of severe sepsis and shock1 for case inclusion‼ Sepsis 2 Definitions Apply

– SIRS, Sepsis, Severe Sepsis, Septic Shock

‼ Sepsis can be determined in the chart without explicit physician diagnosis Any mention of presumed or diagnosed infection and the presence of ≥ 2 SIRS criteria Any noted hypotension or lactate > 2, respiratory or renal failure, thrombocytopenia, etc. =

severe sepsis or shock.’ If severe sepsis or septic shock exists, case reviewed for ‘bundle compliance’ & reported to

CMS

✳ Upshot: document clearly why you suspect or do not suspect an infection. Account for all organ system lab abnormalities Underlying chronic kidney disease? Alternate explanation for elevated bilirubin or low

platelets? Explain why you don’t think organ dysfunction labs are due to sepsis in your note

1. Sep‐1 Early Management Bundle, Severe Sepsis/Septic Shock: v5.3a Measure Updates – 1.5 C.E. Last updated February 27, 2018. Accessed  https://www.qualityreportingcenter.com/wp‐content/uploads/2018/02/IQR_slides_Sepsis_v5.3a_20180227_vFINAL508.pdf September 1, 2018

Sep-1: Sepsis Bundle Treatment3 hours from Sepsis Time Zero

✅Obtaining blood cultures prior to antibiotics✅Broad spectrum antibiotics within 1 hour✅Serum lactate measurement✅Fluid resuscitation (for lactate ≥ 4, hypotension)

6 hours from Sepsis Time Zero✅Repeat serum lactate level (if initial >2 mmol/L)✅Volume status and tissue perfusion bedside

reassessment✅Vasopressors if indicated

2. The Joint Commission. Specifications manual for national hospital inpatient quality measures. 2018. Accessed at https://www.jointcommission.org/specifications_manual_for_national_hospital_inpatient_quality_measures.aspx on 2 September 2018

Page 36: Evaluation and Management of Sepsis/Septic Shock in Adults ... › dam › AAFP › documents › events › fmx › hand… · Evaluation and Management of Sepsis/Septic Shock in

36

Sep-1: Sepsis Bundles

A Word about Reassessment:

✳ Before 6 hours from Time Zero has elapsed✅ After “adequate” (30 mL/kg) fluid resuscitation

✅ Dynamic fluid assessment passive leg raise versus

trial bolus2

✅ Updated vitals, perfusion, exam reviewed

✅ Repeat assessment must be documented

1. Sep‐1 Early Management Bundle, Severe Sepsis/Septic Shock: v5.3a Measure Updates – 1.5 C.E. Last updated February 27, 2018. Accessed  https://www.qualityreportingcenter.com/wp‐content/uploads/2018/02/IQR_slides_Sepsis_v5.3a_20180227_vFINAL508.pdf September 1, 20182. Monnett X, Marik P, Teboul JL. Passive leg raising for predicting fluid responsiveness: a systematic review and meta‐analysis. Intensive Care Med. 2016;42(12):1935‐1947

AES Question 4a.A 35 year-old Hispanic woman called into your clinic nurse describing fevers and nausea, and was directed to the emergency department. You are on call for the clinic that night when the Emergency room physician calls to admit the same patient for pyelonephritis, and mentions to you that the patient meets sepsis criteria.Which of the following is not a part of the initial Sepsis bundle?

A. 2 sets of Blood culturesB. Serum LactateC. Complete Blood Count (CBC)D. Broad Spectrum antibiotics

Page 37: Evaluation and Management of Sepsis/Septic Shock in Adults ... › dam › AAFP › documents › events › fmx › hand… · Evaluation and Management of Sepsis/Septic Shock in

37

Question 4a.A 35 year-old Hispanic woman called into your clinic nurse describing fevers and nausea, and was directed to the emergency department. You are on call for the clinic that night when the Emergency room physician calls to admit the same patient for pyelonephritis, and mentions to you that the patient meets sepsis criteria.Which of the following is not a part of the initial Sepsis bundle?

A. 2 sets of Blood culturesB. Serum LactateC. Complete Blood Count (CBC)D. Broad Spectrum antibiotics

Evaluating Components of SEP-1/SSC Bundles

Page 38: Evaluation and Management of Sepsis/Septic Shock in Adults ... › dam › AAFP › documents › events › fmx › hand… · Evaluation and Management of Sepsis/Septic Shock in

38

Blood Cultures

• Retrospective study of New York State Department of Health following legal mandate of early identification1

– ED triage only, 3 hr bundle initiated within 6 hrs of triage, 3 hrbundle completed within 12 hrs

– 49,331 eligible patients – Results: Each additional hour delay in time to drawing blood

cultures OR 1.04 (95% CI: 1.02 – 1.06) of in-hospital mortality

✳ Conclusion: Draw blood cultures ASAP, but don’t delay antibiotic administration by more than 1 hour

1. Seymour CW, Gesten F, Prescott HC, et al. Time to Treatment and Mortality during Mandated Emergency Care for Sepsis. N Engl J Med. 2017;376(23):2235‐2244. doi:10.1056/NEJMoa1703058.

Antibiotic Selection

✳ Surviving Sepsis Campaign recommends empiric ”broad spectrum antibiotics”

✳ Broad Spectrum monotherapy = gram positive ➕ gram negative coverage

Monotherapy examples: Ceftriaxone, cefotaxime, cefepime, moxifloxacin, levofloxacin, amoxicillin/clavulanate, ampicillin/sulbactam, piperacillin/tazobactam, meropenem Please note exceptions to this: next slide

✳ If concern for MDRO, or in Septic Shock, start with empiric coverage for both Pseudomonas a. and MRSA

1. Rhodes A, Evans L, Alhazzani W. Surviving Sepsis Campaign: International Guielines for Management of Sepsis and Septic Shock: 2016. Crit Care Med. 2017;45(3);486‐552

Page 39: Evaluation and Management of Sepsis/Septic Shock in Adults ... › dam › AAFP › documents › events › fmx › hand… · Evaluation and Management of Sepsis/Septic Shock in

39

Microbiology by pathogen

• Escherichia coli 12%1 / 16.0% • Streptococcus spp. 10.3%1 / 4.1%• Coagulase negative Staphylococcus spp. 9.7%1 / 10.8%• Staphylococcus aureus 9.0%1 / 20.5% • Pseudomonas spp. 2.07%1 / 19.9%• Bacillus spp. 2.07%1 / ---• Enterococcus spp. 2.07%1 / 10.9%• Enterobacter spp. 2.07%1 / 7.0%• Klebsiella spp. <1%1 / 12.7%• Acinetobacter spp. --- / 8.8%• Candida spp. <1%1 / 17.0%• Aspergillus --- / 1.4%• Anaerobes --- / 4.5%• Parasites --- / 0.7%

1. Novosad SA, Sapiano MR, Grigg C, et al. Vital Signs: Epidemiology of Sepsis: Prevalence of Health Care Factors and Opportunities for Prevention.  MMWR Morb Mortal Wkly Rep 2016;65:864–8692. Vincent JL, Rello J, Marshall J, et al. EPIC II Group of Investigators. International study of the prevalence and outcomes of infection in intensive care units. nJAMA. 2009;302(21):2323‐9

CDC / EPIC IIPathogen isolates

Antibiotic Selection• Antibiotics that must be used in combination to pass Sep-1:

Group 1 + Group 2

Aminoglycosides  Or + Clindamycin IV  Or

Ciprofloxacin  Or + Penicillins Or

Aztreonam + Macrolides  Or

Cephalosporins (1st or 2nd generation)  Or

Daptomycin  Or

Linezolid  Or

Glycopeptides

Page 40: Evaluation and Management of Sepsis/Septic Shock in Adults ... › dam › AAFP › documents › events › fmx › hand… · Evaluation and Management of Sepsis/Septic Shock in

40

Early Antibiotics – Large Surviving Sepsis Dataset

Ferrer study, Critical Care Medicine 2014

• 17,990 patients across 165 hospitals with severe sepsis and septic shock.

✳ Linear increase in mortality with every hour delay of antibiotic administration after the first hour.1

1. Ferrer R, Martin-Loeches I, Phillips G, et al. Empiric antibiotic treatment reduces mortality in severe sepsis and septic shock from the first hour: results from a guideline-based performance improvement program. Crit Care Med. 2014;42(8):1749-1755.

• Systematic Review & Meta-Analysis, 20151

• ED triage only, 16,178 patients (14,639 from Ferrer study)

– Antibiotic administration <3 hrs or >3 hrsof triage, did not improve mortality

– Pooled OR 1.16 (95% CI: 0.92 -1.46, p = 0.21)

• Antibiotic administration <1hr or >1hr of severe sepsis/shock recognition, did not improve mortality

– Pooled OR 1.46 (95% CI: 0.89 -2.40, p = 0.13)

• Limitations2

– 7/18 trials excluded due to lack of author response to inquiry

– No consideration of antibiotic selection

Antibiotic Timing

• Systematic Review & Meta-Analysis, 20174

– Primarily ED triage, includes newer studies than previous meta-analysis

• Antibiotic administration <1 hrs or >1 hrs of evaluation, did improve mortality

– Pooled OR 0.67 (95% CI: 0.59 – 0.75)

• As in the previous meta-analysis, this study includes results from Ferrer study. However, after excluding the two largest studies (including the previously discussed Ferrer study), results significantly in favor of early antibiotic administration

– Pooled OR 0.70 (95% CI: 0.57 – 0.87)

• Limitations– Included more settings than just ED triage

1. Sterling SA, Miller WR, Pryor J, Puskarich MA, Jones AE. The Impact of Timing of Antibiotics on Outcomes in Severe Sepsis and Septic Shock: A Systematic Review and Meta-Analysis. Crit Care Med. 2015;43(9):1907-19152. Sivapalan P, Stæhr Jensen J-U. Timing of Therapy in Sepsis: Are Early Antibiotics Important for Prognosis? Crit Care Med. 2015;43(9):2030-2031. 3. Youkee D, Hulme W, Roberts T, Daniels R, Nutbeam T, Keep J. Time Matters: Antibiotic Timing in Sepsis and Septic Shock. Crit Care Med. 2016;44(10):e1016-e1017. 4. Johnston ANB, Park J, Doi SA, et al. Effect of Immediate Administration of Antibiotics in Patients With Sepsis in Tertiary Care: A Systematic Review and Meta-analysis. Clin Ther. 2017;39(1):190-202.e196.

Page 41: Evaluation and Management of Sepsis/Septic Shock in Adults ... › dam › AAFP › documents › events › fmx › hand… · Evaluation and Management of Sepsis/Septic Shock in

41

Antibiotic Timing

• Retrospective Study of New York State Department of Health following legal mandate of early identification1

– ED triage only, 3 hr bundle initiated within 6 hrs of triage, 3 hrbundle completed within 12 hrs

– 49,331 eligible patients Each additional hour to completion of 3 hr bundle OR 1.04 (95% CI:

1.02 – 1.05) of in-hospital mortality Each additional hour to administration of antibiotics OR 1.04 (95% CI:

1.03 – 1.06) of in-hospital mortality Compared to bundled completion <3 hrs to >3 hrs, 14% greater

odds of in-hospital mortality Difference greater in patients with vasopressor use OR 1.05 (95% CI:

1.03-1.07), suggesting potential greater benefit in septic shock

1. Seymour CW, Gesten F, Prescott HC, et al. Time to Treatment and Mortality during Mandated Emergency Care for Sepsis. N Engl J Med. 2017;376(23):2235‐2244. doi:10.1056/NEJMoa1703058.

Antibiotic Timing

• Retrospective Study of New York State Department of Health following legal mandate of early identification1

– ED triage only, 3 hr bundle initiated within 6 hrs of triage, 3 hrbundle completed within 12 hrs

– 49,331 eligible patients Each additional hour to completion of 3 hr bundle OR 1.04 (95% CI:

1.02 – 1.05) of in-hospital mortality Each additional hour to administration of antibiotics OR 1.04 (95% CI:

1.03 – 1.06) of in-hospital mortality Compared to bundled completion <3 hrs to >3 hrs, 14% greater

odds of in-hospital mortality Difference greater in patients with vasopressor use OR 1.05 (95% CI:

1.03-1.07), suggesting potential greater benefit in septic shock

1. Seymour CW, Gesten F, Prescott HC, et al. Time to Treatment and Mortality during Mandated Emergency Care for Sepsis. N Engl J Med. 2017;376(23):2235‐2244. doi:10.1056/NEJMoa1703058.

Overall, trend favors early antibiotic administration

Effect size of early antibiotic administration may depend on degree of delay

Larger number of patients with septic shock would be required to properly power an RCT

Page 42: Evaluation and Management of Sepsis/Septic Shock in Adults ... › dam › AAFP › documents › events › fmx › hand… · Evaluation and Management of Sepsis/Septic Shock in

42

Antibiotic De-escalation

✳Infectious Disease Society of America, CDC as well asSurviving Sepsis Campaign, SCCM, ESCIM advocate: daily antibiotic assessment for de-escalation1,2,3

⬇CDC recommends “Antibiotic Time Out” < 48 hours of start⬇Daily assessment of antibiotics⬇Narrow spectrum based on cultures, suspicion for infectious agent⬇Decrease length of treatment, depending on infection type, site

1. Barlam T, Cosgrove S, Abbo L. Implementing an Antibiotic Stewardship Program: guidelines by the Infectious Disease Society of America and the Society for Healthcare Epidemiology of America. Clin Infect Dis. 2016;62(10):e51‐e772. Centers for Disease Control and Prevention. “Core Elements of Hospital Antibiotic Stewardship Programs.” last updated 23 February 2017. Accessed https://www.cdc.gov/antibiotic‐use/healthcare/implementation/core‐elements.html 2 September 20183. Howell M, Davis A. Management of Sepsis and Septic Shock. JAMA. 2017;317(8):847‐848

Crystalloid Fluids✳The SSC goal is ≥ 30 milliliters/kilogram of

crystalloid✅Recommended administration in sepsis (3) with

hypoperfusion✅Determines septic shock after ‘adequate fluid

resuscitation’❓ Limited evidence behind the dosing and breadth

of applicability, acknowledged by SSC, driven by ‘usual practice’1

❓ Conflicting studies looked at mortality and benefit derived from following fluid resuscitation with mixed mortality results2,3

1. Rhodes A, Evans L, Alhazzani W. Surviving Sepsis Campaign: International Guielines for Management of Sepsis and Septic Shock: 2016. Crit Care Med. 2017;45(3);486‐5522. Seymour C, Gesten F, Prescott H, et al. Time to Treatment and Mortality during Mandated Emergency Care for Sepsis. N. Eng J Med. 2017;376:2235‐22443. Maitland K, Kiguli S, Opoka RO, et al. FEAST Trial Group. Mortality after fluid bolus in African children with severe infection. N Engl J Med. 2011;364:2483095

Page 43: Evaluation and Management of Sepsis/Septic Shock in Adults ... › dam › AAFP › documents › events › fmx › hand… · Evaluation and Management of Sepsis/Septic Shock in

43

Timing of Fluid Resuscitation

• Retrospective study of New York State Department of Health following legal mandate of early identification1

– ED triage only, 3 hr bundle initiated within 6 hrs of triage, 3 hr bundle completed within 12 hrs

– 49,331 eligible patients – Multivariate model

Each additional hour delay in time to initial IV fluid bolus OR 1.01 (95% CI: 0.99 –1.02) of in-hospital mortality

❓ Conclusion: The only component in sepsis bundle not associated with improved mortality was time until IV fluid bolus administration

1. Seymour CW, Gesten F, Prescott HC, et al. Time to Treatment and Mortality during Mandated Emergency Care for Sepsis. N Engl J Med. 2017;376(23):2235‐2244. doi:10.1056/NEJMoa1703058.

✳2016 SSC recommendation✳Fluid resuscitation based on

responsiveness✅ CVP low predictive value for assessing

fluid responsiveness1

✅ Passive leg raising (PLR) predicts response of cardiac output to further volume expansion2

Examiner moves the patient’s legs from semi-recumbent or horizontal to 45°2

Simulates 500 mL bolus✅ Fluid bolus trial✅ Serum lactate clearance✅ Point of Care ultrasound

Central venous assessment

Fluid Reassessment

1. Eskesen TG, Wetterslev M, Perner A. Systemic review including re‐analysis of 1148 individual data sets of central venous pressure as a predictor of fluid responsiveness. Intensive Care Med. 2016;42(3):324‐322. Monnett X, Marik P, Teboul JL. Passive leg raising for predicting fluid responsiveness: a systematic review and meta‐analysis. Intensive Care Med. 2016;42(12):1935‐19473. Illustration taken from Figure  in Marik PE, Monnet X, Teboul JL Hemodynamic parameters to guide fluid therapy. Ann Intensive Care. 2011;1(1). Copyright Policy –Open Access.License accessed https://creativecommons.org/licenses/by/2.0/ on 2 September 2018

Page 44: Evaluation and Management of Sepsis/Septic Shock in Adults ... › dam › AAFP › documents › events › fmx › hand… · Evaluation and Management of Sepsis/Septic Shock in

44

✳SCC 2016 recommendation:

✳Conclusion: Monitor lactate clearance to guide fluid resuscitation and/or vasopressor administration

Lactate Clearance Substitute for ScvO2

• 2004 Critical Care Medicine prospective observational study, multivariate logistic regression of variables

– 111 patients, comparison of lactate clearance within 6 hours and mortality

– 38.1% +/- 34.6% lactate clearance for survivors and 12.0% +/- 51.6% lactate clearance for non-survivors

In multivariate logistic regression, lactate clearance inversely related with mortality (p = 0.04)

• 2010 JAMA randomized trial comparing ScvO2 guided therapy to at least 10% lactate clearance.

– Trial within the EGDT framework, so CVP and MAP included prior to consideration of ScvO2 or lactate clearance.

– 17% mortality with lactate clearance guided therapy and 23% mortality with ScvO2 guided therapy.

Septic Shock patients had similar in-hospital mortality whether managed via lactate clearance or ScvO2 normalization

1. Nguyen HB, Rivers EP, Knoblich BP, et al. Early lactate clearance is associated with improved outcome in severe sepsis and septic shock. Crit Care Med. 2004;32(8):1637-1642.2. Jones AE, Shapiro NI, Trzeciak S, et al. Lactate clearance vs central venous oxygen saturation as goals of early sepsis therapy: a randomized clinical trial. JAMA. 2010;303(8):739-746..

• Previously products such as hydroxyethyl starch used alternately to crystalloid for resuscitation

– Previously recommended by SSC– Large molecules suspended in crystalloid

• Examples: starch, dextran, albumin, fresh frozen plasma, gelatin

– Faster intravascular volume expansion– $$$– Historically, allergenic, concern for mortality

✳ SSC 2016 recommendation:✳ Conclusion: Starches no longer

recommended resuscitation per SSC 2016 guidelines

– Avoid hydroxyethyl starch colloids1

Colloid Fluids• 2015 FISSH Intensive Care Medicine systematic

review & meta-analysis. 10 studies, 6664 patients2

– Focused on sepsis patients– Risk of renal replacement therapy: starch >

crystalloid– Trials heterogenous -- imprecise comparisons2

• 2018 Cochrane review; colloids vs crystalloids in critically ill patients. 69 studies, n=30,0003

– No evidence of difference in mortality (relative risk: 0.97, 95% CI 0.86-1.09) at follow-up, 30 and 90 days

– No increased risk of allergic reaction• Less itching, fewer rashes with crystalloid

– Non-significant increase in blood transfusion, renal replacement therapy with starch group

– Cochrane recommendation not definitive• Several studies in process >> clarity?

1. Rhodes A, Evans L, Alhazzani W. Surviving Sepsis Campaign: International Guielines for Management of Sepsis and Septic Shock: 2016. Crit Care Med. 2017;45(3);486‐5522. Rochwerg B, Alhazzani W, Gibson A. Fluid type and the use of renal replacement therapy in sepsis: a systematic review and network meta‐analysis. Intensive Care Med. 2015;41(9):1561‐713. Lews SR, Pritchard MW, Evans DJ. Co lloids versus crystalloids for fluid resuscitation in critically ill people. Cochrane Database Syst Rev. 2018;8:CD000567

Page 45: Evaluation and Management of Sepsis/Septic Shock in Adults ... › dam › AAFP › documents › events › fmx › hand… · Evaluation and Management of Sepsis/Septic Shock in

45

✳ SSC 2016 recommendation✳ Conclusion: Albumin = second-line

resuscitation when requiring large quantities of crystalloid1

✅ Similar or slightly better mortality rates to crystalloid2,3,4

✅Depends on meta-analysis 3,5

Colloids - Albumin• Systematic review, network meta-analysis, 20145

– 14 studies, 18,916 patients, final outcome of mortality

– Meta-analysis with network design, involving different crystalloids and colloids.

– When compared by groups:• Lower mortality for crystalloids than

starches with high confidence• Lower mortality for albumin than

starches or crystalloids with moderate confidence

– When compared individually:• Lower mortality for albumin than

unbalanced crystalloids and low-molecular weight starches

• Lower mortality for balanced crystalloids than with saline and either low- or high-molecular weight starches

Conclusion: In sepsis, balanced crystalloids or albumin is associated with lower mortality

1. Rhodes A, Evans L, Alhazzani W. Surviving Sepsis Campaign: International Guielines for Management of Sepsis and Septic Shock: 2016. Crit Care Med. 2017;45(3);486-5522. Lews SR, Pritchard MW, Evans DJ. Co lloids versus crystalloids for fluid resuscitation in critically ill people. Cochrane Database Syst Rev. 2018;8:CD0005673. Patel A, Laffan M, Waheed U, Brett S. Randomised trials of human albumin for adults with sepsis: systematic review and meta-analysis with trial sequential analysis of all-cause mortality. BM. 2014; ;349:g45614. Caironi P, Tognoni G, Masson S, et al. Albumin replacement in patients with severe sepsis or septic shock. N Engl J Med. 2014;370(15):1412-215. Rochwerg B, Alhazzani W, Sindi A, et al. Fluid resuscitation in sepsis: a systematic review and network meta-analysis. Annals of Internal Medicine. 2014;161(5):347-355..

Blood Transfusions

• Prospective parallel-group trial, ICU patients with septic shock, 998 patients, at a hemoglobin of 9 or less1

– Randomized to transfusion with 1 unit leukoreduced red blood cells at a hemoglobin of 7 or 9.

– No difference in mortality or any of the secondary morbidity outcomes

– Lower threshold group had less transfusions

✳SSC 2016 recommendation: transfusion of red blood cells (RBC) when hemoglobin < 7.0 g/dL in absence of myocardial ischemia, severe hypoxia, hemorrhage

✳Conclusion: No mortality difference in septic shock with a lower hemoglobin threshold of 7 compared to 9.

1. Holst LB, Haase N, Wetterslev J, et al.TRISS Trial Group Scandinavian Critical Care Trials Group. Lower versus higher hemoglobin threshold for transfusion in septic shock. N Engl J Med. 2014;371(15):1381-1391. doi:10.1056/NEJMoa1406617.

Page 46: Evaluation and Management of Sepsis/Septic Shock in Adults ... › dam › AAFP › documents › events › fmx › hand… · Evaluation and Management of Sepsis/Septic Shock in

46

AES Question 4b.A 35 year-old Hispanic woman called into your clinic nurse describing fevers and nausea, and was directed to the emergency department. You are on call for the clinic that night when the Emergency room physician calls to admit the same patient for pyelonephritis, and mentions to you that the patient meets sepsis criteria.What is the time frame you have to give initial broad spectrum antibiotics?

A. 10 minutesB. 1 hourC. 3 hoursD. 6 hours

Question 4b.A 35 year-old Hispanic woman called into your clinic nurse describing fevers and nausea, and was directed to the emergency department. You are on call for the clinic that night when the Emergency room physician calls to admit the same patient for pyelonephritis, and mentions to you that the patient meets sepsis criteria.What is the time frame you have to give initial broad spectrum antibiotics?

A. 10 minutesB. 1 hourC. 3 hoursD. 6 hours

Page 47: Evaluation and Management of Sepsis/Septic Shock in Adults ... › dam › AAFP › documents › events › fmx › hand… · Evaluation and Management of Sepsis/Septic Shock in

47

Source control✳2016 SSC recommendation1

✳ Rapid diagnosis of anatomic involvement

✳ Infection site source controlDrainage, resection, debridement, device

removalASAP delay affects survival2

In unstable septic patient, medical stabilization may require source control first1

1.Rhodes A, Evans L, Alhazzani W. Surviving Sepsis Campaign: International Guielines for Management of Sepsis and Septic Shock: 2016. Crit Care Med. 2017;45(3);486‐5522. AZuhata T, Kinoshita K, Kawano D, et al. Time from admission to initiation of surgery for source control is a critical determinant of survival in patients with gastrointestinal perforation with associated septic shock. Crit Care. 2014;18(30:$87

• Serum procalcitonin (PCT) is a peptide, calcitonin prehormone

– Bacterial infection stimulates increased expression of the gene

• Severity of infection, mortality correlates with level of procalcitonin

• Procalcitonin’s role in infection unclear– Half-life: 25-30 hours– Measurement: quantitative homogenous assay2

✳ IDSA suggests ICU implementation of ‘serial’ procalcitonin measurements.

✳ SSC 2016 guidelines recommendation✳ Conclusion: Utilize procalcitonin in sepsis care

✅ Utilize to decrease antibiotic utilization1

✅ Monitoring for PCT clearance most useful

Procalcitonin• 2013 Lancet meta-analysis3

– 30 studies included, 3244 critically ill patients –early diagnosis of sepsis versus inflammation without infection

– Procalcitonin Sensitivity 0.77, specificity 0.79, AUC 0.85 (95% CI, 0.81-0.88)

Recommendation: utilize PCT as a marker but cannot definitively diagnose sepsis

• 2015 PLOS One Systematic review, meta-analysis4

– 23 studies, 3,994 patients with sepsis– Findings: elevated PCT associated with higher

risk of mortality, Relative risk 2.60, 95% CI (2.05-3.30). Sensitivity 0.76 and specificity 0.64

– Initial PCT levels = limited in prognosis but PCT “non-clearance” = prognostic; Higher sensitivity 0.72 and specificity 0.77

1.  Barlam T, Cosgrove S, Abbo L. Implementing an Antibiotic Stewardship Program: guidelines by the Infectious Disease Society of America  and the Society for Healthcare Epidemiology of America. Clin Infect Dis. 2016;62(10):e51‐e772. Jin M, Khan A. Procalcitonin: uses in the clinical laboratory for the diagnosis of sepsis. Laboratory Medicine. 2010; 41(3):173‐1773. Wacker C, Prkno A, Brunkhorst FM, Schlattmann P. Procalcitonin as a diagnostic marker for sepsis: a systematic review and meta‐analysis. Lancet Infect Dis. 2013;13(5):426‐354. Liu D, Su L, Han G, Yan P, Xie L. Prognostic value of procalcitonin in adult patients with sepsis: a systematic review and meta‐analysis. PLoS One. 2015; 10(6): e0129450

Page 48: Evaluation and Management of Sepsis/Septic Shock in Adults ... › dam › AAFP › documents › events › fmx › hand… · Evaluation and Management of Sepsis/Septic Shock in

48

✳ SSC & Sepsis 3 indications: Sepsis with persistent hypotension or lactate > 2 after adequate fluid resuscitation

✳ Conclusion: maintain MAP target of ≥ 65 mmHg

✳ Norepinephrine first, then add vasopressin (up to 0.03 units/minute), then epinephrine1

✳ Dopamine restricted to select patient group: ✅ Septic shock with absolute or relative

bradycardia✅ Low risk of tachyarrhythmia

✳ Dobutamine reserved for patients with persistent hypoperfusion after adequate fluids and vasopressors

Vasopressors & Inotropes1

Multiple single-center trials compared MAPs>65 to MAPs>85

– Resulted in an increase in cardiac index2,3

– No clear effect on lactate levels, urine output, oxygen consumption2,3

– Multi-center, open label trial: No difference in 28 day mortality or 90 days5

– Higher MAP target associated with risk of arrhythmia5,6

– MAP 80-85 trend: less renal replacement therapy for patients with chronic hypertension5

– Patients older than 75 tolerated lower MAP of 65-70 mmHg better5

2016 Cochrane review of 26 RCTs, comparing 6 vasopressors in 12 comparisons7

Dopamine had more arrhythmias than norepinephrine; ”might increase mortality”

“No evidence of substantial differences in mortality” among the vasopressors evaluated

1. Rhodes A, Evans L, Alhazzani W, et al.Surviving Sepsis Campaign: International Guidelines for Management of Sepsis and Septic Shock:2016. Crit Care Med.2017;45(3).486‐5522. LeDoux D, Astiz ME, Carpati CM, Rackow EC. Effects of perfusion pressure on tissue perfusion in septic shock. Crit Care Med. 2000;28(8):2729‐323. Bourgoin A, Leone M, Delmas A, Garnier F, Albanese J, Martin C. Increasing mean arterial pressure in patients with septic shock: effects on oxygen variables and renal function. Crit Care Med. 2005;33(4):780‐64. Thooft A, Favory R, Salgado DR, et al. Effects of changes in arterial pressure on organ perfusion during septic shock. Crit Care 2011;15(5):R2225. Asfar P, Meziani F, Hamel JF, et al. High versus low blood‐pressure target in patients with septic shock. N Engl J Med. 2014;370(17):1583‐936. Lamontagne F, Meade MO, Hebert PC, et al. Higher versus lower blood pressure targets for vasopressor therapy in shock: multicenter pilot randomized controlled trial. Intensive Care Med. 2016;42(4):542‐5507. Gamper G, Havel C, Arrich J, et al. Vasopressors for hypotensive shock. Cochrane Database Syst Rev. 2016;2:CD003709

✳SSC 2016 recommendation✳Conclusion: hemodynamic

instability despite “adequate fluid resuscitation + vasopressor therapy”1

✅ Do not use to prevent shock✅ Do not routinely test for adrenal

insufficiency✅ Hydrocortisone 200 mg IV daily dose,

tapered• 3-7 days: taper once vasopressors

are not indicated1

• Avoid fludricortisone, dexamethasone (HPAA suppression)

Steroids 2008 CORTICUS trial: improved resolution

of septic shock, SOFA scores2

Mortality benefit debatable – present in meta-analysis2,3,4,5

– Mortality benefit: CORTICUS trial4and 2018 ADRENAL trial5 >> Cochrane Review

– Reduction in 28-day mortality by 13%4

– Dose dependent benefit: low dose, longer (RR0.87, 95% CI 0.78-0.97)

ICU length of stay reduced by 2 days4

– Improved recovery from septic shock by day 73,4

Risks of Hyperglycemia, Hypernatremia Risks from infection, gastrointestinal

bleeds, neurologic weakness not significant4

1. Rhodes A, Evans L, Alhazzani W. Surviving Sepsis Campaign: International Guielines for Management of Sepsis and Septic Shock: 2016. Crit Care Med. 2017;45(3);486‐5522. Sprung CL, Annane D, Keh D, et al. Hydrocortisone therapy for patients with septic shock. N Engl J Med. 2008;358(2):111‐243. Sligl W, Milner D, Sundar S, et al. Safety and efficacy of corticosteroids for the  treatment of septic shock: a systematic review and meta‐analysis. Clin Infect Dis. 2009;49:93‐1014. Annane D, Bellissant E, Bollaert PE, et. Al. Corticosteroids for treating sepsis (Review). Cochrane Database Syst Rev. 2015; 12:CD0022435. Venkatesh B, Finfer J, Cohen D, et al. ADRENAL trial group. Adjunctive glucocorticoid therapy in patients with septic shock. 2018;378(9):797‐808

Page 49: Evaluation and Management of Sepsis/Septic Shock in Adults ... › dam › AAFP › documents › events › fmx › hand… · Evaluation and Management of Sepsis/Septic Shock in

49

Advanced respiratory support✳SSC recommendations: Indicated in septic shock,

respiratory failure, Acute Respiratory Distress Syndrome (ARDS)✅Trial CPAP (continuous positive airway pressure) ventilation first

for added PEEP✳SSC 2016 recommendations for mechanical

ventilation:✅Tidal volumes 6mL/kg (not higher)✅Peak plateau pressure < 30 cm H2O✅High PEEP recommended✅No evidence behind using any single ventilator mode✅For maximal oxygenation, consider neuromuscular blockade for

total paralysis✅For ARDS patients, caution with fluid administration

1.Rhodes A, Evans L, Alhazzani W. Surviving Sepsis Campaign: International Guielines for Management of Sepsis and Septic Shock: 2016. Crit Care Med. 2017;45(3);486‐552

Advanced respiratory support✳SSC recommendations: Indicated in septic shock,

respiratory failure, Acute Respiratory Distress Syndrome (ARDS)✅Trial CPAP (continuous positive airway pressure) ventilation first

for added PEEP✳SSC 2016 recommendations for mechanical

ventilation:✅Tidal volumes 6mL/kg (not higher)✅Peak plateau pressure < 30 cm H2O✅High PEEP recommended✅No evidence behind using any single ventilator mode✅For maximal oxygenation, consider neuromuscular blockade for

total paralysis✅For ARDS patients, caution with fluid administration

1.Rhodes A, Evans L, Alhazzani W. Surviving Sepsis Campaign: International Guielines for Management of Sepsis and Septic Shock: 2016. Crit Care Med. 2017;45(3);486‐552

A larger literature conversation is beyond the scope of this presentation

Page 50: Evaluation and Management of Sepsis/Septic Shock in Adults ... › dam › AAFP › documents › events › fmx › hand… · Evaluation and Management of Sepsis/Septic Shock in

50

¿More focused treatment?‼ Despite hundreds of trials no sepsis-specific treatment

consistently effective from 20 plus years of research1

Examples: interleukin 1, interleukin 6, tumor necrosis factor, rhAPC,2,3,4 Granulocyte colony-stimulating factor,5,6 Intravenous Immune globulins,7 among others

Challenges:• Difficulty to streamline, standardize care during trials• Heterogeneity of presentation in patients and infectious agents• Varying organ system involvement with sepsis presentations1

1. Cohen J, Vincent JL, Adhikari N, Machado F, et al. Sepsis: a roadmap for future research. The Lancet Infectious Disease Commission. 2015; 15(5): P581‐6142. Bernard GR, Vincent JL, Laterre PF, et al. Efficacy and safety of recombinant human activated protein C for severe sepsis. N Engl J Med. 2001. 8;344(10):699‐7093. Ranieri VM, Thompson BT, Barie PS, et all. Drotecogin alfa (activated) in adults with septic shock. N. Engl J Med. 2012;366(22):2055‐644. Marti‐Carvajal AJ, Sola I, Lathyris D, Cardona AF. Human recombinant activated protein C for severe sepsis. Cochrane Database Syst Rev. 2012; 14(3):CD0043885. Stephens D, Thomas J, Higgins A, et al. Randomized, double‐blind, placebo‐controlled trial of granulocyte colony‐stimulating factor in patients with septic shock. Crit Care Med. 2008;36(2):448‐546. Bo L, Wang F, Zhu J, Li J, Deng X. Granulocyte‐colony stimulating factor (G‐CSF) and granulocyte‐macrophage colony stimulating factor (GM‐CSF) for sepsis a meta‐analysis. Crit Care. 2011;15(1):R58 7. Dellenger RP, Levy M, Rhodes A, et al. Surviving Sepsis Campaign: International Guidelines for Management of Severe Sepsis and Septic Shock: 2012. Crit Care Med. 2013;41(2):580‐637

🚧Microfluidic Assay – diagnose sepsis based on neutrophil movement1

🚧Blood culture Assays – Rapid blood culture pathogen diagnosis already in use in select hospitals. (97.5% sensitivity and 99.3% specificity)2

🚧 New discoveries 🚧🚧 Endothelial barrier-targeted drugs3

• Multiple preclinical, early clinical trials

• Focus on endothelial junction, membrane stabilization.

• Focus on restriction of cytokine, neutrophil activation

🚧 Epithelial barrier-targeted drugs3

• Multiple trials in preclinical and pilot studies

• Focus on regulation of microbiome, affect gut permeability

1. Abbasi J. A Microfluidic Assay to Diagnose Sepsis. JAMA. 2018;319(18):18532. Voelker R. Detecting Bacteremia Pathogens. JAMA. 2017;317(13):13083. .Cohen J, Vincent JL, Adhikari N, Machado F, et al. Sepsis: a roadmap for future research. The Lancet Infectious Disease Commission. 2015; 15(5): P581‐614

Page 51: Evaluation and Management of Sepsis/Septic Shock in Adults ... › dam › AAFP › documents › events › fmx › hand… · Evaluation and Management of Sepsis/Septic Shock in

51

🚧 New discoveries: treatment 🚧

🚧 2017 Thiamine + Vitamin C + steroids study – Virgina1

– Retrospective before-after study: 7 month control with subsequent 7 month treatment.– Vitamin C 6 grams/day divided 4x daily, hydrocortisone 50 mg every 6 hours x 7 days

and thiamine (Vitamin B1) 200 mg every 12 hours for 4 days – all given intravenously < 24 hours after ICU admission

– Hospital Mortality 8.5% in treatment group, 40.4% in control group (p<0.001)– SOFA score decreased in all treatment patients– Mean vasopressor treatment length 18.3 hours in treatment group, 54.9 in control group

🚧 2018 Thiamine Study – University of Kentucky2

– Retrospective study, 369 patients in septic shock– IV thiamine administration < 24 hours after admission – Thiamine dose 500 mg IV Q 8 hours x 72 hours– Improved lactate clearance (HR1.307, 95% CI 1.002-1.704)

– Reduction in 28-day mortality (HR 0.666, 95% CI, 0.4990-0.905)

– No effect on rates of kidney injury, SOFA scores or ventilator/vasopressor-free days

1. Marik PE, Khangoora V, Rivera R, Hooper MH, Catravas J. Hydrocortisone, Vitamin C, and Thiamine for the Treatment of Severe Sepsis and Septic Shock: A retrospective before‐after study. Chest. 2017; 151(6):1229‐1238 2.Woolum J, Abner E, Kelly A, et al. Effect of Thiamine Administration on Lactate Clearance and Mortality in Patients with Septic Shock. Crit Care Med. 2018; volume online first(p).

AES Question 5You admitted your clinic patient from question 3, the 67 year-old African American man with a history of hypertension, diabetes mellitus type 2, and tobacco use, diagnosing him with pneumonia. He quickly developed sepsis, then septic shock, requiring 2-day ICU stay and vasopressors. When he is stabilized, you accept him back to the hospital’s intermediate care unit to continue treatment.

What is essential for sepsis care at this point?A. Diuresis after fluid resuscitationB. Consultation with nutritionC. De-escalation of antibioticsD. Consultation for speech therapy

Page 52: Evaluation and Management of Sepsis/Septic Shock in Adults ... › dam › AAFP › documents › events › fmx › hand… · Evaluation and Management of Sepsis/Septic Shock in

52

Question 5You admitted your clinic patient from question 3, the 67 year-old African American man with a history of hypertension, diabetes mellitus type 2, and tobacco use, diagnosing him with pneumonia. He quickly developed sepsis, then septic shock, requiring 2-day ICU stay and vasopressors. When he is stabilized, you accept him back to the hospital’s intermediate care unit to continue treatment.

What is essential for sepsis care at this point?A. Diuresis after fluid resuscitationB. Consultation with nutritionC. De-escalation of antibioticsD. Consultation for speech therapy

Treatment SummaryBritish Medical Journal Interactive Sepsis Care1

1. Gotts J, Matthay M. Sepsis: pathophysiology and clinical management. BMJ. 2016;353:i1585

Page 53: Evaluation and Management of Sepsis/Septic Shock in Adults ... › dam › AAFP › documents › events › fmx › hand… · Evaluation and Management of Sepsis/Septic Shock in

53

Can’t see the Forest for the Trees?

Current Sepsis ‘Usual Care’

✳ Broad spectrum antibiotics within an hour✳ Blood cultures before antibiotics but < 1 hour ✳ Fluid status assessment

✳ Lactate, blood pressure monitoring

✳Dynamic fluid status reassessment < 6 hours

✳ Admit to ICU for vasopressors, advanced respiratory support: persistent hypotension or acute respiratory failure/ARDS

Family Medicine –Trained Hospitalist Involvement in Sepsis

Page 54: Evaluation and Management of Sepsis/Septic Shock in Adults ... › dam › AAFP › documents › events › fmx › hand… · Evaluation and Management of Sepsis/Septic Shock in

54

Local involvement in Sepsis• Attend your hospital’s Sepsis Quality Meetings

1. Provide your physician/clinician voice & experience2. If your hospital does not have a sepsis committee, start one3. Work with your hospital abstractor to review how they abstract

cases

4. Focus on improving ’low hanging fruit’ in Sepsis care Educate physicians: ordering serum lactate Educate nurses, physicians: fluids, correct antibiotics Educate nurses: blood cultures before antibiotics Work with pharmacy: streamline rapid antibiotics

administration

Local involvement in Sepsis• Sepsis Committee: beyond the basics

1. Investigate and improve the hospital’s response to sepsis Electronic health record alerts: Accuracy?Code Sepsis: Is there infrastructure or support? Identify and empower your team, the stakeholders

2. Utilize abstracted cases to identify focused sepsis care improvement projects

3. Provide hospital or region-wide Sepsis awareness education

Page 55: Evaluation and Management of Sepsis/Septic Shock in Adults ... › dam › AAFP › documents › events › fmx › hand… · Evaluation and Management of Sepsis/Septic Shock in

55

Future Focus🚧 Patient Care: Focus on Quality of life, Post-Sepsis Rehabilitation, Support1

SSC THRIVE Initiative Focus on Rehabilitation

Therapy within 90 days associated with lower 10-year mortality risk1

🚧 Sepsis Diagnosis: Identify specific biomarkers, tailor treatment to specific organ dysfunction2

Microvascular hemodynamic status analysis

🚧 Research: Develop better end points, primary outcomes Focus on patient-centered outcomes: quality of life markers, long-term survival2

Investigate use of an ‘adaptive trial’ model along with or in addition to randomized control trials2

• Allowing for targeted, personalized therapy2

• Modifying dosing, randomization, exclusion/inclusion throughout the study Utilize In-silico-based individual modelling and treatment specification3

1. Prescott HC, Angus DC. Enhancing Recovery From SepsisA Review. JAMA. 2018;319(1):62–75.2. Cohen J, Vincent JL, Adhikari N, Machado F, et al. Sepsis: a roadmap for future research. The Lancet Infectious Disease Commission. 2015; 15(5): P581‐6143. Namas R, Zamora R, Namas R, An G et al. Sepsis: something Old, Something New, and a Systems View. J Crit Care. 2012; 27(3): 314.e1–314.e11

How to improve your sepsis management

1 Understand the significant conceptual and treatment debates ongoing with Sepsis✅ Sepsis 2 versus Sepsis 3✅ Sepsis care protocols and what those mean for clinicians

2 Elevate your clinic and hospital vigilance –‘think sepsis.’✅ Realize that you, the primary care provider, are often the first identifier of sepsis✅ Utilize clinic or hospital resources to identify sick patients and escalate treatment✅ Participate in peri-Sepsis care: vaccination, patient education, chronic disease management

3 Name and implement the goals of treatment for sepsis ✅ Early broad-spectrum antibiotic✅ Focused fluid resuscitation, utilizing dynamic reassessment of responsiveness✅ In a fluid-non-responsive state vasopressors✅ De-escalation of antibiotic therapy ✅ Awareness of bundle therapy and its role in hospitals, public health, and your practice

Page 56: Evaluation and Management of Sepsis/Septic Shock in Adults ... › dam › AAFP › documents › events › fmx › hand… · Evaluation and Management of Sepsis/Septic Shock in

56

Questions